Вы находитесь на странице: 1из 77
el 8 PERFORMANCE BOOSTERS TO CONQUER ANY LAW EXAM (NOT JUST THE BAR EXAM) SIEGFRED B. MISON, Esa. Philippine Copyright, 2008 sineinep MISON ISBN 978-971-23-5187-7 No portion of this book may be copied or reproduced in books, pamphlets, outlines or notes, whether printed, mimeographed, typewrit- ten, copied in different electronic devices or in any other form, for distribution or sale, without the written permission of the author except brief passages in books, articles, reviews, legal papers, and judicial or other official proceedings with proper citation. ‘Any copy of this book without the corresponding number and the signature of the author on this page either proceeds from an illegitimate source or is in possession of one ‘who has no authority to dispose of the same. ALL RIGHTS RESERVED BY THE AUTHOR No___0666_ Printed by REX pRiNTING Company, INC. sypognaphy & creative lithogeaphy (4? Fowrin Cue ty Telos Tast0i-7124108 DEDICATION To those who will make use of this book, heed the words of advice of your mentors and professors for they know better ~ at least for the time being. ‘To those who will write questions in any law exam, fear now — for the “mystery” behind your questions shall soon be revealed in this book. To those who will check the answers in any law exam, rejoice — for the checking shall soon be effortless and cease to be a headache. And to those who have given me the inspiration, I whole- heartedly dedicate this simple “booster” manual. 8/16 FOREWORD Ever since I joined the academe, I have met so many kinds of law students. Most of them are scared out of their wits every time they take an exam. Some of them are confident but they still end up with a so-so grade in the exam. Some of them study their hearts out but their exam grades do not reflect the amount of work they put into their preparation. If you are one of these students, then this booster book is for you! Law students, regardless of which school they earned their law degree from, are more or less equal when it comes to their “knowledge of the law.” The greatest disparity among law students that I have taught is found in their ability to articu- late what they know into writing. This booster book authored by my fellow professor, Atty. ‘Mison, is one of the many useful books that law students MUST read and imbibe. The experience of Atty. Mison as a student in taking and passing law exams (including the Philippine Bar and even the California Bar) represents a diverse approach in dealing with a time-pressured exam. Further, the experience of Atty. Mison as a professor in preparing and checking law ‘exams gives the reader an insight worth knowing to “conquer any law exam.” Ultimately, the principles in this booster book extend to any law practitioner whose objective is to write pleadings that contain a very organized, logically presented, and legally accurate arguments. I salute Atty. Mison for this commendable work. Dean ‘San Sebastian College of Law FOREWORD Ever since I joined the academe, I have met so many kinds of law students. Most of them are scared out of their wits, every time they take an exam. Some of them are confident but they still end up with a so-so grade in the exam. Some of them study their hearts out but their exam grades do not reflect the amount of work they put into their preparation. If you are one of these students, then this booster book is for you! Law students, regardless of which school they earned their law degree from, are more or less equal when it comes to their “knowledge of the law.” The greatest disparity among law students that I have taught is found in their ability to articu- late what they know into writing. ‘This booster book authored by my fellow professor, Atty. ‘Mison, is one of the many useful books that law students MUST read and imbibe. The experience of Atty. Mison as a student in taking and passing law exams (including the Philippine Bar ‘and even the California Bar) represents a diverse approach in dealing with a time-pressured exam. Further, the experience of Atty. Mison as a professor in preparing and checking law exams gives the reader an insight worth knowing to “conquer any law exam.” Ultimately, the principles in this booster book extend to any law practitioner whose objective is to write pleadings that contain a very organized, logically presented, and legally accurate arguments. I salute Atty. Mison for this commendable work. SVs Sarre Atty. Willard B. Riano Dean ‘San Sebastian College of Law FOREWORD ‘Atty. Siegfred B. Mison’s new magnum opus is a familiar topic that is brought to fore by a learned and practical author whose objective is to alleviate the plight not only of the Bar ean- didates but also of the Professors and Bar examiners. Indeed, the result of the 2007 Bar Examinations is more than a wake up call to all members of the legal community. The work Atty. ‘Mison had put into this booster indicates his rich experience as fa conscientious academic mentor & scholar; studious drill mas- ter and a fierce competitor in sports and in the field of law. ‘While it is true that the study of law is aimed to produce good and principled lawyers who would protect and defend ‘their clients to the best of their capabilities, it is equally true that passing the Bar examinations is a mandatory requirement in order to realize the said objective. Certainly, in this booster, the secrets of how to take the Bar exams are now in the open, up for grabs by any Bar candi- date who needs all the confidence booster he can get before tak- ing the most grueling examinations. While it is no guarantee that reading this booster will assure the Bar candidate that he will pass the Bar examinations, it will definitely improve his preparations and bring his confidence level to a new high. I know the amount of work, time, patience and passion for excellence that was put into this work of the author and for all these, Atty. Mison should be highly commended for bringing forth what was once discussed exclusively by Review schools ‘and for unselfishly sharing so much of himseff, his talents and skills with the legal community. Atty. Plaridel J. Partner Bohol Bohol Jimenez Law Office Professor of Law San Beda College of Law University of the East College of Law FOREWORD Thave a friend in law school who took the bar examination ayear ahead of me. Unfortunately, he did not pass the bar. He tried to appear unaffected by it, but truly, I could feel his pain deep inside. He kept himself busy by teaching in a university and doing several other things. Two years later, he took the bar exams for the second time. He failed again. This time around, my friend got himself re-assigned to a university in Mindanao, leaving in Manila his wife and two small daughters. Perhaps my friend wanted some quiet time and space for a while. Three years later, my friend took the bar exams again. For the third time, he didn't make it. A little over two years later, Iheard my friend and his wife separated. According to his wife, my friend was never able to recover from his failed attempts to pass the bar examinations. It affected him mentally, emotionally and psychologically. ‘Truly, the bar exams can destroy many things a plan, a career, a relationship, a family, and even a life. I do not know how many plans, relationships, families, or lives have been ruined by the bar exams. There are no statis- tics available. But every year, on the average, only around 20% of those who take the bar exams pass it. Roughly, that trans- lates to only about a thousand of the more than 5,000 who take the bar exams annually. It is in this context that I highly recommend this book. I firmly believe there are only two (2) essential require- ‘The first is knowledge ments for a candidate to pass the bar. of the law. is ‘The second is knowledge of the techniques for an- ‘apps has quaclona peony Like Siegfred Mison, I firmly believe that the secor as important as the first. Your know!- edge of the law is nothing if you do not know when and how to apply it. This book will definitely provide you with the right tech- niques for passing the bar. Take time to read it especially be- fore plunging into a tedious bar review. It would take you only a day to finish and understand the 8 principles in this book. It is my wish that this book will be able to help many candidates pass the bar. It is my wish that this book will be able to save a lot of pains and troubles. It is my wish that this book will be able to save a lot of plans, relationships, families, and lives. My highest praise to the author for sharing his precious time, talent and treasure to help others attain their own dream in life - which is to become a lawyer — like him. Atty. L. Lacson National Bestselling Author “12 Little Things Filipinos Can Do To Help Our Country” Partner Malcolm Law Offices INTRODUCTION Based on the results of the 2007 Philippine Bar examina- tions, we can speculate on two things. First, the standards of the examiners (as well as the Bar Chairman) of developing the 2007 Bar questions and checking the answers perhaps were simply too high. Second, since only 5% of the examinees ob- tained a passing grade of 75%, the passing grade mandated under the existing Rules of Court, perhaps the quality of law graduates has deteriorated. It had to take an unprecedented move by the Supreme Court to lower the passing grade from 75 to 70 just to accommodate a higher number of Bar passers this year. What made the 2007 Bar examinations more difficult to ass as compared to previous Bar examinations? Was it the questions, the strict checking, or the quality of students? This, ‘book does not aim to provide the reasons whatever they may be. The fact remains that law students and/or Bar candidates now and in the future will always have difficulty in passing any law examination. This book aims to provide exam-taking techniques or “boosters” to enable the examinee to perform better in any law examination, Just like an athlete in any competitive sport, the physical and mental attributes of the student should be at its peak every time he/she has to take an exam, Manny Pacquiao or Tiger Woods did not become champions by simply being gift- ed or talented in their craft. They had to put in a lot of hours to train before a big fight/game and develop the ability to shift their skills into high gear upon command. Ihave taken the Bar examination twice. The first time ‘was in 1996 in Manila and the second was in 2005 in California. My preparations in both were similar — regimented, focused, and geared towards developing peak performance at a certain time. I virtually isolated myself for 5 months for the Philippine Bar and for 6 weeks for the California Bar as I strategized my xi approach to these examinations. My desire in this book is to share what I did that helped me perform better during the four Sundays of September in 1996 and during the three consecu- tive days of the last week of February 2005. Some of the suggested performance boosters in this book may have already been said by professors and done by examin- es in the past. However, when taken as whole and internalized to the fullest, examinees in a law exam should have the ability to have the fighting heart of a Pacquiao or the killer instincts of ‘Tiger Woods to complete the mission on any given day. 8 PERFORMANCE BOOSTERS (TO CONQUER ANY LAW EXAM) TABLE OF CONTENTS Ist Booster: Fear is not a Factor! (Conquer your fear of law exams 2nd Booster: Beat the Clock! (Manage your time effectively.) ‘3rd Booster: Dissect the Essay! (Know the “Call of the Question”) 4th Booster: Spot the Issue; Win Half the Battle! (Some Proven Methods to Locate Issue(s] in a Problem) 5th Booster: Make a Road Map. (An outline organizes your answer. Translation: Maximum Points) 6th Booster: Write a Masterpiece! (A well-written work makes the answer easier to understand and check) ‘7th Booster: Know the Enemy! (Having a crystal bowl won't hurt.) 8th Booster: Conquer All Others! *Summary *Bonus: Sample Question, Analysis and Answer 1st Booster: Fear is not a Factor! (Conquer your fear of law exams) Law school professors are generally characterized as in- timidating, daunting, and at times even unreasonable. I think some professors of freshmen law students are given the man- date — To instill fear! Let the weak in spirit quit early, let the strong remain and try to survive the grueling legal education. Law students encounter professors who “torture” them with punishing recitation sessions and seemingly unanswerable examination questions. Regardless of their preparation for a Jaw school exam, law students almost always find their efforts inadequate in coming up with the correct answers. This is con- firmed when after the examination, the professor gives several possible answers to the questions which leaves the students more baffled about how they fared in their exams. Bottom line, professors including myself would say that it is how you argue ‘your answer and not the answer itself that will give you the highest credit. This makes sense since even the Justices in the highest court would inevitably have different ways of respond- ing to an issue presented by a case. Law students, particularly freshmen, are expectedly uptight, anxious and probably even terrified when they center the law school. In preparing for an exam, students will typically try to get each and every available reference material — textbooks, their own notes, their classmate’s notes, study aids produced by a study group, study aids produced by a study group from another school, memory aids, past exams, ete. One can only imagine the extent of their photocopying budget. One of my advantages of being a working student was that I was earning enough to support an unpredictable photocopying budget for law school. ‘Students will then try to read, some even twice, all those materials they gathered. Of course, the more they read, the 1 2 8 PERFORMANCE BOOSTERS T0 CONQUER ANY LAW EXAM more they know. Unfortunately, the more they know, the more they forget. A human brain can only absorb, process and retain s0 much information. And even if they do remember everything they read, they are unable to correlate such information with the question in the actual exam. This is because of the intimidation factor. Tonce had a student who confessed that she was petrified at the mere sight of me. And quite a few of her classmates shared the same feeling. She said it was something in me that even without uttering a single word, she was already intimidated. (Probably my demeanor or my reputation in law school) Nonetheless, she told me that she really studied all the reading assignments and allotted a significant amount of time in my class compared to her other classes. However, when called for recitation, she told me later than as soon as she stood up, she forgot everything she read the night before. She knew the answer I assume, but she barely made sense. She tried to regain her composure. She could not. With all her preparation, she still got a failing mark for her incoherent answer. Ah, another vietim by the wayside! On the last day of regular classes where most professors will tell their students the coverage of the exams, I give some parting words to my students who I may have ‘terrified.’ I tell my class that as a law school professor I play a role to insure that they get quality legal training. Training not education Tsay. Any person can read and perhaps understand any law textbook, even a caveman with a good memory can memorize the Bill of Rights verbatim. But for me, the most important skill that any law student can acquire in law school is the ability to think and act under pressure. The profession will expose these future lawyers to several pressing cases where they will be tested on how they can react quickly and accurately. A law ‘school exam is just one of the tools through which students can acquire this skill. A good professor should have this in mind. However, for the most part, students are simply intimidated. How can fear be removed or at least mitigated? Fear is a psychological reaction of people to a certain stimuli. At examination time, law students typically enter the room with so much fear. Fear of not knowing the answer to the ST BOOSTER: FEAR IS NOT A FACTOR! 3 (Conquer your fear of law exams) question; fear of forgetting what they studied; fear of flunking the exam and even fear of expulsion from the school. I say that there is a ready antidote for such fear — preparedness in mind, body, and spirit. ‘To be mentally prepared, one can simply study all the reading assignments given throughout the course and pay attention to the lectures and class discussions. It can’t be that hard since that is part of being a law student. You pay tuition fees to attend your classes and read your books. ‘To have your body physically prepared for an exam is easier. Just make sure you are well rested before each exam. After an intense review session, relax. Of course, a little exercise won't hurt, Science has proven that an increase in blood circulation thru exercise will always help the brain to function better. ‘Tohave that spirit to conquer your fear, you need to psyche yourself up for the exam. Your psychological mindset before any exam will definitely affect how well you will perform: In golf, how you approach the game dictates how you score at the end of the day. Prior to any tournament, a golfer checks the wind conditions, studies the fairway and pin placement, and formulates an overall game plan even before his first swing. ‘Tiger Woods, maybe the greatest golfer of all time, would always say that he plays in a tournament to win, nothing else. It is not out of sheer audacity that he says this but simply out of well-placed confidence in his game. He is confident that so long as he puts in the time to practice, he ean score well in any given day. You too can score well in any exam as long as you ut in the time to be prepared. Preparation for an exam will give you that confidence. Those fears of forgetting and failing will be negated by how ‘you approach any given exam. Believe that you will pass, and ‘you will. As long as you studied what you were supposed to study, there is nothing to fear. Of course, the approach as recommended in this book should help, I hope. Remember, a law school exam is not designed to intimidate students. It is formulated by professors who were once students. ‘There is no secret code to break to answer their questions. 4 8 PERFORMANCE BOOSTERS TO CONQUER ANY LAW EXAM Ist Booster TALK: In an article entitled “Smart Fear,” known psychotherapist George Hartwell’ claimed that “top performance can be fueled by fear.” Below are some of the Hartwell techniques to convert the “energy of fear from nervousness to calm energy.” I simply revised the “mantra” to make them more applicable to exam taking. a. Take Time Out as in taking a sip of water, a deep breath, or even just stare at the ceiling when faced with a difficult problem. “Relax.” &. Give up trying to control outcomes. The preparation art is as important as the execution part — “I can let go.” c. Prayer: You have done what you can so at the start of the exam, you can say “God you are in control.” d. Convert fear into a brief anger fit. “Focus, damn in” Get excited ~ “I love a challenge from Professor X~his exam brings out the best in me.” Use the gift of humor. “What’s the fuss, answering this exam is even more difficult than answering questions from my girlfriend?” g. Be confident. Remember your success image. “Nothing can stop me from topping this exam!” ‘Mentally cue yourself: “This will be my defining moment — Let's get it on!” "George Hartwell i a Christian Counselor, a personal therapist, marriage and family counselor based in Toronto, Canada. 2nd Booster: Beat the Clock! (Manage your time effectively) If you studied diligently, nothing stands in your way of passing the exam except panic. Paniccan easily be removed by effective time management. Law school exams are expectedly subject to immense time pressure. After all, stability under pressure is a trademark of all lawyers. Thus, part of the legal education is a written examination which tests the ability of the students to read and analyze a fact pattern and to apply whatever theoretical knowledge they learned to a specific issue with time constraints. ‘Time allocation for the entire exam and for each question is therefore crucial to optimize your scores. Upon receipt of the questionnaire, the very first thing any examinee should do is to glance at the point allocation for each question. If the questions are given the same weight, then the time allotted per question is length of the exam in minutes divided by the number of questions. For a ten question final exam with a two hour time limit, you should allot twelve ‘minutes per question. If the questions are not evenly weighted, then allocate more minutes to those questions with higher point allocation. It appears to be common sense but a lot of students upon receiving the exam questions just jump on the very first one instinctively! I did when I was a freshman in law school. + Some professors specify the points allotted for each issue, ‘some don't. In case the points are not indicated, then allot more time to those issues that are highly contentious. Always keep in mind that no matter how well you answer a specific issue in any given question, you will only get the points allotted to that issue at best. Thus, spend more time analyzing those issues that you think the professor will appreciate a comprehensive discussion on and spend less time on “minor” details. Of course, being able to answer all the issues will give you bonus points; 5 6 _ 8 PERFORMANCE BOOSTERS TO CONQUER ANY LAW EXAM. but that is a different matter. Time limitations should always tell a prudent student to focus on the bigger picture. Once you getithe time allocation forthe entire examination, you should adhere to it as much as possible. I had the chance of checking a final examination where the student simply answered, “Sir, I did not have enough time but the answer is yes.” Without any supporting discussion, that answer will hardly get any credit. His answer to the last question in that same exam was “Yes, because of Article 777 of the Civil Code.” When I looked at his answers to the other questions, they were intelligently written, well-thought of, and needless to say, deserved maximum credit. However, on the whole, he barely passed that exam simply because of his poor time management. After spending about a minute allotting your time per question, you will now need to allocate that time within the question itself, as in how much time would you spend in reading the problem, writing your answer, etc. recommend a time allocation per question broken down as follows: 20% read and understand the problem; 20% analyze the issues; 20% draft an outline paying close attention to the issues; 30% write your answer making sure key words are included; 10% review what you wrote. Resist the temptation of writing an answer right away. ‘Think about your answer. A well-thought of answer gets the maximum credit. 2nd Booster EXERCISE: Below is the 2006 Philippine Bar exam questions in Civil Law. As an exercise, don't read the question in its entirety but i ion. Allot the time per question then make a mental estimate of the time you will be spending in reading the problem, analyzing the issues, drafting an outline, writing your answer, and even reviewing what you ‘wrote. This exercise will somehow give you an idea how easy it, is to manage the time in answering any law examination. 2ND BOOSTER: BEAT THE CLOCK! 7 (Manage your time effectively) hes Under Article 213 of the Family Code, no child under 7 years of age shall be separated from the mother unless the court finds compelling reasons to order otherwise. 1. Explain the rationale of this provision. 2.5% 2. Give at least 3 examples of “compelling reasons” which Justify the taking away from the mother’s custody of her ‘child under 7 years of age. 2.5% a Saul, a married man, had an adulterous relation with Tessie. In one of the trysts, Saul’s wife, Cecile, caught them in fagrante. Armed with a gun, Cecile shot Saul in a fit of extreme Jealousy, nearly killing him. Foun (4) years after the incident, ‘Saul filed an action for legal separation against Cecile on the ground that she attempted to kill him. 1. If you were Saut’s counsel, how will you argue his case? 25% 2. If you were the lawyer of Cecile, what will be your defense? 25% 3. If you were the judge, how will you decide the case? 5% -m- Ed and Beth have been married for 20 years without children. Desirous to have a baby, they consulted Dr. Jun Canlas, a prominent medical specialist on human fertility He advised Beth to undergo artificial insemination. It was found that Ed's sperm count was inadequate to induce pregnancy. Hence, the couple looked for a willing donor. Andy, the brother of Ed, readily consented to donate his sperm. After a series of tests, Andy's sperm was medically introduced into Beth's ovary She became pregnant and 9 months later, gave birth to a baby boy, named Alvin. 1. Whois the father of Alvin? Explain. 2.5% 6 8 PERFORMANCE BOOSTERS TO CONQUER ANY LAW EXAM but that is a different matter. Time limitations should always tell a prudent student to focus on the bigger picture. ‘Once you get the timeallocation for the entire examination, you should adhere to it as much as possible. I had the chance of checking a final examination where the student simply answered, “Sir, I did not have enough time but the answer is yes.” Without any supporting discussion, that answer will hardly get any credit. His answer to the last question in that same exam was “Yes, because of Article 777 of the Civil Code.” When I looked at his answers to the other questions, they were intelligently written, well-thought of, and needless to say, deserved maximum credit. However, on the whole, he barely passed that exam simply because of his poor time management. After spending about a minute allotting your time per question, you will now need to allocate that time within the question itself, as in how much time would you spend in reading the problem, writing your answer, ete. I recommend a time allocation per question broken down as follows: 20% read and understand the probler 20% analyze the issues; 20% draft an outline paying close attention to the issues; 30% write your answer making sure key words are included; 10% review what you wrote. Resist the temptation of writing an answer right away. Think about your answer. A well-thought of answer gets the maximum credit. 2nd Booster EXERCISE: Below is the 2006 Philippine Bar exam questions in Civil Law. As an exercise, don’t read the question in its entirety but focus on the number of questions and the point allocation for each question. Allot the time per question then make a mental estimate of the time you will be spending in reading the problem, analyzing the issues, drafting an outline, writing your answer, and even reviewing what you wrote. This exercise will somehow give you an idea how easy it is to manage the time in answering any law examination. 2ND BOOSTER: BEAT THE CLOCK! 7 (Manage your time effectively) alii Under Article 213 of the Family Code, no child under 7 ears of age shall be separated from the mother unless the court finds compelling reasons to order otherwise. 1. Explain the rationale of this provision. 2.5% 2. Give at least 3 examples of “compelling reasons” which Justify the taking away from the mother's custody of her child under 7 years of age. 2.5% one Saul, a married man, had an adulterous relation with Tessie. In one of the trysts, Saul’s wife, Cecile, caught them in flagrante, Armed with a gun, Cecile shot Saul in a fit of extreme Jealousy, nearly killing him. Foun (4) years after the incident, Saul filed an action for legal separation against Cecile on the ground that she attempted to kill him. 1. If you were Saul’s counsel, how will you argue his case? 25% 2. Ifyou were the lawyer of Cecile, what will be your defense? 2.5% 3. If you were the judge, how will you decide the case? 5% —m~ Ed and Beth have been married for 20 years without children. Desirous to have a baby, they consulted Dr. Jun Canlas, a prominent medical specialist on human fertility He advised Beth to undergo artificial insemination. It was found that Ed’s sperm count was inadequate to induce pregnancy. Hence, the couple looked for a willing donor. Andy, the brother of Ed, readily consented to donate his sperm. After a series of tests, Andy's sperm was medically introduced into Beth's ovary She became pregnant and 9 months later, gave birth to a baby boy, named Alvin. 1. Whois the father of Alvin? Explain. 2.5% 8 _ 8PERPORMANCE BOOSTERS TO CONQUER ANY LAW EXAM What are the requirements, if any, in order for Ed to establish his paternity over Alvin. 2.5% eos Gigi and Ric, Catholics, got married when they were 18 years old. Their marriage was solemnized on August 2, 1989 by Ric’s uncle, a Baptist Minister, in Calamba, Laguna. He overlooked the fact that his license to solemnize marriage expired the month before and that the parties do not belong to his congregation. After 5 years of married life and blessed with 2 children, the spouses developed irreconcilable differences, 80 they parted ways. While separated, Ric fell in love with Juliet, a 16 year-old sophomore in a local college and a seventh-Day Adventist. They decided to get married with the consent of Juliet’s parents. She presented to him a birth certificate showing she is 18 years old. Ric never doubted her age much less the authenticity of her birth certificate. They got married in a Catholic church in Manila. A year after, Juliet gave birth to twins, Aissa and Aretha. 1. What is the status of the marriage between Gigi and Ric = valid, voidable or void? Explain. 2.5 % 2, What is the status of the marriage between Ric and Juliet — valid, voidable or void? Explain. 2.5% 3. Suppose Ric himself procured the falsified birth certificate to persuade Juliet to marry him despite her minority and assured her that everything is in order. He did not divulge to her his prior marriage with Gigi. What action, if any, can Juliet take against him? Explain. 2.5% 4, If you were the counsel for Gigi, what action/s will you take to enforce and protect her interests? Explain. 2.5% -V- ‘Spouses Bong and Linda wanted to sell’ their house. They found a prospective buyer, Ray. Linda negotiated with Ray for the saie of the property. They agreed on a fair price of P2 Million. Ray sent Linda .a letter confirming his intention to buy the property. Later, another couple, Bernie and Elena, offered a 2ND BOOSTER: BEAT THE CLOCK! 9 (Manage your time effectively) similar house at a lower price of PL.5 Million. But Ray insisted ‘on buying the house of Bong and Linda for sentimental reason. Ray prepared a deed of sale to be signed by the couple and a ‘manager's check for P2 Million. After receiving the P2 Million, Bong signed the deed of sale. However, Linda was not able to sign it because she was abroad. On her return, she refused to sign the document saying she changed her mind. Linda filed suit for nullification of the deed of sale and for moral and exemplary damages against Ray. 1. Will the suit prosper? Explain. 2.5% 2. Does Ray have any cause of action against Bong and Linda? Can he also recover damages from the spouses? Explain, 25% =v Gemma filed a petition for the declaration of nullity of her ‘marriage with Arnell on the ground of psychological incapacity -She alleged that after 2 months of their marriage, Arnell showed signs of disinterest in her, neglected her and went abroad. He returned to the Philippines after 3 years but did not even get in touch with her. Worse, they met several times in social functions but he snubbed her. When she got sick, he did not visit her even if he knew of her confinement in the hospital. Meanwhile, Arnell ‘met an accident which disabled him from reporting for work and earning a living to support himself. Will Gemma’s suit prosper? Explain. 5% — va Marvin, a Filipino, and Shelley, an American, both residents of California, decided to get married in their local Parish, Two years after their marriage, Shelley obtained a divorce in California. While in Boracay, Marvin met Manel, @ Filipina, who was vacationing there. Marvin fell in love with her. After a brief, courtship and complying with all the requirements, they got married in Hongkong to avoid publicity, it being Marvin's second marriage. Is his marriage to Manel valid? Explain. 5% 8 8 PERFORMANCE BOOSTERS TO CONQUER ANY LAW EXAM 2, What are the requirements, if any, in order for Ed to establish his paternity over Alvin. 2.5% So RPS Gigi and Ric, Catholics, got married when they were 18 ‘years old. Their marriage was solemnized on August 2, 1989 by Ric's uncle, a Baptist Minister, in Calamba, Laguna. He overlooked the fact that his license to solemnize marriage expired the month before and that the parties do not belong to his congregation. After 5 years of married life and blessed with 2 children, the spouses developed irreconcilable differences, s0 they parted ways. While separated, Ric fell in love with Juliet, a 16 year-old sophomore in a local college and a seventh-Day Adventist. They decided to get married with the consent of Juliet’s parents. She presented to him a birth certificate showing she is 18 years old. Ric never doubted her age much less the authenticity of her birth certificate. They got married in a Catholic church in Manila. A year after, Juliet gave birth to twins, Aissa and Aretha. 1. What is the status of the marriage between Gigi and Ric — valid, voidable or void? Explain. 2.5 % 2. What is the status of the marriage between Ric and Juliet — valid, voidable or void? Explain. 2.5% 3. Suppose Ric himself procured the falsified birth certificate to persuade Juliet to marry him despite her minority and assured her that everything is in order. He did not divulge to her his prior marriage with Gigi. What action, if any, can Juliet take against him? Explain. 2.5% If you were the counsel for Gigi, what action /s wild you take to enforce and protect her interests? Explain. 2.5% ey ‘Spouses Bong and Linda wanted to sell’ their house. They found a prospective buyer, Ray. Linda negotiated with Ray for the sale of the property. They agreed on a fair price of P2 Million. Ray sent Linda .a letter confirming his intention to buy the property. Later, another couple, Bernie and Elena, offered a 2ND BOOSTER: BEAT THE CLOCK! 9 (Manage your time effectively) similar house at a lower price of Pl.5 Million. But Ray insisted on buying the house of Bong and Linda for sentimental reason. Ray prepared a deed of sale to be signed by the couple and a ‘manager's check for P2 Million. After receiving the P2 Million, Bong signed the deed of sale. However, Linda was not able to sign it because she was abroad. On her return, she refused to sign the document saying she changed her mind. Linda filed suit for nullification of the deed of sale and for moral and exemplary damages against Ray. 1. Will the suit prosper? Explain. 2.5% 2. Does Ray have any cause of action against Bong and Linda? Can he also recover damages from the spouses? Explain. 25% ae Gemma filed a petition for the declaration of nullity of her ‘marriage with Arnell on the ground of psychological incapacity ‘She alleged that after 2 months of their marriage, Arnell showed signs of disinterest in her, neglected her and went abroad. He returned to the Philippines after 3 years but did not even get in touch with her. Worse, they met several times in social functions but he snubbed her. When she got sick, he did not visit her even if he knew of her confinement in the hospital. Meanwhile, Arnell ‘met an accident which disabled him from reporting for work and earning a living to support himself. Will Gemma’s suit prosper? Explain. 5% vn Marvin, a Filipino, and Shelley, an American, both residents of California, decided to get married in their local parish. Two years after their marriage, Shelley obtained a divorce in California. While in Boracay, Marvin met Manel, @ Filipina, who was vacationing there. Marvin fell in love with her. After a brief, courtship and complying with all the requirements, they got married in Hongkong to avoid publicity, it being Marvin's second marriage. Is his marriage to Manel valid? Explain. 5% 8 8 PERFORMANCE BOOSTERS T0 CONQUER ANY LAW EXAM 2. What are the requirements, if any, in order for Ed to establish his paternity over Alvin. 2.5% aoa: Gigi and Ric, Catholics, got married when they were 18 years old. Their marriage was solemnized on August 2, 1989 by Ric's uncle, a Baptist Minister, in Calamba, Laguna. He overlooked the fact that his license to solemnize marriage expired the month before and that the parties do not belong to his congregation. After 5 years of married life and blessed with 2 children, the spouses developed irreconcilable differences, so they parted ways. While separated, Ric fell in love with Juliet, a 16 year-old sophomore in a local college and a seventh-Day Adventist. They decided to get married with the consent of Juliet’s parents. She resented to him a birth certificate showing she is 18 years old. Ric never doubted her age much less the authenticity of her birth certificate. They got married in a Catholic church in Manila. A year after, Juliet gave birth to twins, Aissa and Aretha. 1. What is the status of the marriage between Gigi and Ric — valid, voidable or void? Explain. 2.5 % 2. What is the status of the marriage between Ric and Juliet — valid, voidable or void? Explain. 2.5% 3. Suppose Ric himself procured the falsified birth certificate to persuade Juliet to marry him despite her minority and ‘assured her that everything is in order. He did not divulge to her his prior marriage with Gigi. What action, if any, can Juliet take against him? Explain. 2.5% 4. If you were the counsel for Gigi, what action! will you take to enforce and protect her interests? Explain. 2.5% ye ‘Spouses Bong and Linda wanted to sell’ their house. They found a prospective buyer, Ray. Linda negotiated with Ray for the sale of the property. They agreed on a fair price of P2 Million. Ray sent Linda .a letter confirming his intention to buy the property. Later, another couple, Bernie and Elena, offered a 2ND BOOSTER: BEAT THE CLOCK! 9 (Manage your time effectively) similar house at a lower price of P15 Million. But Ray insisted ‘on buying the house of Bong and Linda for sentimental reason. Ray prepared a deed of sale to be signed by the couple and a manager's check for P2 Million. After receiving the P2 Million, Bong signed the deed of sale. However, Linda was not able to sign it because she was abroad. On her return, she refused to ‘sign the document saying she changed her mind. Linda filed suit for nullification of the deed of sale and for moral and exemplary damages against Ray. 1. Will the suit prosper? Explain. 2.5% 2 Does Ray have any cause of action against Bong and Linda? Can he also recover damages from the spouses? Explain. 2.5% Savi Gemma filed a petition for the declaration of nullity of her marriage with Arnell on the ground of psychological incapacity She alleged that after 2 months of their marriage, Arnell showed signs of disinterest in her, neglected her and went abroad. He returned to the Philippines after 3 years but did not even get in touch with her. Worse, they met several times in social functions but he snubbed her. When she got sick, he did not visit her even if he knew of her confinement in the hospital. Meanwhile, Arnell met an accident which disabled him from reporting for work and earning a living to support himself. Will Gemma’s suit prosper? Explain. 5% ~via Marvin, a Filipino, and Shelley, an American, both residents of California, decided to get married in their local parish. Two years after their marriage, Shelley obtained a divoree in California. While in Boracay, Marvin met Manel, @ Filipina, who was vacationing there. Marvin fell in love with her. After a brief, courtship and complying with all the requirements, they got married in Hongkong to avoid publicity, it being Marvin's second marriage. Is his marriage to Manel valid? Explain. 5% 10 8 PERFORMANCE BOOSTERS T0 CONQUER ANY LAW EXAM = Vi - Alberto and Janine migrated to the United States of America, leaving behind their 4 children, one of whom is Manny. ‘They own a duplex apartment and allowed Manny to live In one of the units. While in the United States, Alberta died. His widow and all his children executed an Extrajudicial Settlement of Alberto’s estate wherein the 2-door apartment was assigned by all the children to their mother, Janine. Subsequently, she sold the property to George. The latter required Manny to sign a prepared Lease Contract so that he and his family could continue occupying the unit. Manny refused to sign the contract alleging that his parents allowed him and his family to continue occupying the premises. If you were George's counsel, what legal steps will you take? Explain. 5% ae A drug lord and his family reside in a small bungalow where they sell shabu and other prohibited drugs. When the police found the illegal trade, they immediately demolished the house because according to them, it was a nuisance per se that should be abated. Can this demolition be sustained? Explain. 5% Sire Don died after executing a Last Will and Testament leaving his estate valued at P12 Million to his common-law wife Roshelle. He is survived by his brother Ronie and his half- sister Michelle. 1. Was Don's testamentary disposition of his estate in accordance with the law on succession? Whether you agree or not, explain your answer. 2.5% 2. If Don failed to execute a will during his lifetime, as his Tawyer, how will you distribute his estate? Explain. 2.5% 3. Assuming he died intestate survived by his brother Ronie, his half-sister Michelle, and his legitimate son Jayson, how will you. distribute his estate? Explain. 2.5% ‘2ND BOOSTER: BEAT THE CLOCK! u (Manage your time effectively) 4. Assuming further he died intestate, survived by his father Juan, his brother Ronie, his half-sister Michelle, and his legitimate son Jayson, how will you distribute his estate? Explain. 2.5% —xI- Spouses Alfredo and Racquel were active members of a religious congregation. They donated a parcel of land in favor to that congregation in a duly notarized Deed of Donation, subject to the condition that the Minister shall construct thereon a place of worship within 1 year from the acceptance of the donation. In an affidavit he executed in behalf of the congregation, the Minister accepted the donation. The Deed of Donation was not registered with the Registry of Deeds. However, instead of constructing a place of worship, the Minister constructed a bungalow on the property he used as his residence. Disappointed with the Minister, the spouses revoked the donation and demanded that he vacate the premises immediately. But the Minister refused to leave, claiming that aside from using the bungalow as his residence, he is also using it as a place of worship on special occasions. Under the circumstances, can Alfredo and Racquel evict the Minister and recover possession of the property? If you were the couple's counsel, what action will you take to protect the interests of your clients? 5% —XW— _ Tony bought a Ford Expedition from a car dealer in Muntinlupa City. As payment, Tony issued a check drawn against his current account with Premium Bank. Since he has «@ good reputation the car dealer allowed him to immediately drive home the vehicle merely on his assurance that his check is sufficiently funded. When the car dealer deposited the check, it was dishonored on the ground of “Account Closed”. After an investigation, it was found that an employee of the bank misplaced Tony's account ledger. Thus, the bank erroneously ‘assumed that his account no longer exists. Later, it turned out that Tony's account has more than sufficient funds to cover 128 PERFORMANCE BOOSTERS TO CONQUER ANY LAW EXAM the check. The dealer however, immediately filed an action for recovery of possession of the vehicle against Tony for which he was terribly humiliated and embarrassed. Does Tony have a cause of action against Premium Bank? Explain. 5% — XI — Arturo sold his Pajero to Benjamin for P1 Million. Benjamin took the vehicle but did not register the sale with the Land Transportation Office. He allowed his son Carlos, a minor who did not have a driver's license, to drive the car to buy pan de sal in a bakery .On the way, Carlos, driving in a reckless manner, sideswiped Dennis, then riding a bicycle. As a result, he suffered serious physical injuries. Dennis filed a criminal complaint against Carlos for reckless imprudence resulting in serious physical injuries, 1. Can Dennis file an independent civil action against Carlos and his father Benjamin for damages based on quasi- delict? Explain. 2.5% 2, Assuming Dennis’ action is tenable, can Benjamin raise the defense he is not liable because the vehicle is not registered in his name? Explain. 2.5% —xIV- Zirsthoussous delos Santos filed a petition for change of name with the Office of the Civil Registrar of Mandaluyong City under the administrative proceeding provided in Republic Act No.9048. He alleged that his first name sounds ridiculous and is extremely difficult to spell and pronounce. After complying with the requirements of the law, the Civil Registrar granted his petition and changed his first name Zirxthoussous to “Jesus.” His full name now reads “Jesus delos santos.” Jesus delos santos moved to General Santos City to work ina multi-national company. There, he fell in love and married Mary Grace delos Santos. She requested him to have his first name changed because his new name “Jesus delos Santos” is the same as that of her father who abandoned her family and became a notorious drug lord. She wanted to forget him. Hence, Jesus filed another petition with the Office of the Local Civil 2ND BOOSTER: BEAT THE CLOCK! 18 (Manage your time effectively) Registrar to change his first name to “Roberto.” He claimed that the change is warranted because it will eradicate all vestiges of the infamy of Mary Grace’s father. Will the petition for change of name of Jesus delos Santos to Roberto delos santos under Republic Act No.9048 prosper? Explain. 10% a 1. What entries in the Civil Registry may be changed or corrected without a judicial order? 2.5% 2. Mayan illegitimate child, upon adoption by her natural father, use the surname of her natural mother as her middle name? 2.5% —xvI-— 1. Under Article 2219 of the Civil Code, moral damages may bbe recovered in the cases specified therein, several of which are enumerated below. Choose the case wherein you cannot recover moral damages. Explain. 2.5% ¢ 1. Acriminal offense resulting in physical injuries 2. Quasi-delicts causing physical injuries 8. Immorality or dishonesty 4, Illegal search 5. Malicious prosecution 2. Article 36 of the Family Code provides that a marriage contracted by any party who, at the time of the celebration, was psychologically incapacitated to comply with the essential marital obligations of marriage, shall be void. Choose the spouse listed below who is psychologically incapacitated. Explain. 2.5% 1. Nagger 2. Gayoor lesbian 48 PERFORMANCE BOOSTERS T0 CONQUER ANY LAW EXAM 38. Congenital sexual pervert 4. Gambler Alcoholic NOTHING FOLLOWS. COMMENT: You should not have taken more than two ‘minutes to allot the time per question in this four hour (240 minutes) exam. Below are possible time allocations for this exercise. 1, 240 minutes divided by 16 questions = 15 minutes per question 2, 240 minutes divided by 100 points = 24 minutes per 10 point question, 12 minutes per 5 point question, 6 minutes per 2.5 point question. 3. 240 minutes broken down as 48 minutes (20%) to read and understand the problem; 48 minutes (20%) analyze the issues; 48 minutes (20%) to draft an outline for all the questions; 96 minutes (30%) to write all the answers; and 24 minutes (10%) to review what you wrote. ‘The simplest distribution is Solution 1. Using this gauge, you should also allot the time to be spent per sub-question. ime management is effective if the weight given uub-questions are more or less equal. This will not work if, as in the exam given above, some questions have more sub questions than the others or if some questions have more points than the others. For instance, Question X has 4 sub-questions whereas Question XIV does not have any sub- question. In Question X, if you further subdivide 15 minutes into four parts, as in 3 minutes per sub-question, then you may not have as much time to answer all of them. Meanwhile, you may have over-allotted the whole 15 minutes to answer just ‘one question in Question XIV. I strongly suggest allotting the minutes per question based on the corresponding weight of each as in Solution 2. Spend more time in questions whose point allocations are 2ND BOOSTER: BEAT THE CLOCK! 6 (Manage your time effectively) greater. In the example above, there are four questions with point allocation of ten points (Questions II, IV, X, and XIV) while the remaining twelve questions have point allocations of 5 points each. That being said, allot twice as much minutes to Questions I, IV, X, and XIV compared to the others. It does not have to be a mathematical certainty, a mere approximation will be sufficient in order to come up with an effective time ‘management. Hence, the time distribution will be based on the weight given per question, to include sub-questions. Solution 3 is something complicated and not advisable. I will not suggest that you read the entire exam questions first, analyze each of the issues, make an outline, and then write your answer. You will likely lose focus on each question and you might even forget the important things that you thought of writing when you first read the question. Although writing an outline will enable to you to catch the key words necessary for a good answer, your mind will be so cluttered with all the items that you read in the other questions. After the bigger exercise of allotting the time for the entire exam, make a mental allocation of the time per question. For instance, you can distribute the 15 minutes allotted in Question II (ten point question) into 5 minutes to read and understand the problem and analyze the issues, 2 minutes to draft an outline, 5 minutes to write your answer, and 2 minutes to review what you wrote. Leave one minute extra to breathe or relax before going to the next question. Most, if not all law exams, are time-pressured purposely formulated to test the ability of law students to respond under pressure. Effective time management is just one way to relieve the pressure. ‘Try and allot time per question in all of your exams. Once ‘you get the opportunity to take the Bar exam, time management, will become 2nd nature and you will be able to conquer your fear of not being able to answer all the questions due to lack of time. You will be surprised as you will have plenty of time to spare in answering questions in any law exam if ONLY you can effectively manage the time. 3rd Booster: Dissect the Essay! (Know the “Call of the Question” of the Problem) ‘A majority of law exams entail a complex hypothetical set of facts with intentionally obscured issues which the law student needs to discern. Most professors give a lot of credit if the student is able to spot the issues presented by the question. Even if the answer is wrong or not fully supported by legal basis, half the battle is won. ‘Some exai when you read that 300-500 word narrative, you will already be guided by the question. will have to put in some key words that will inevitably give rrise to a legal issue. After all, cases are filed in our courts due to disputes in the wordings in a contract or in the law. Remember that simple words like “and” and “or” have far reaching legal consequences. Pay attention to dates when the call of the question will make them crucial. For instance, in a bigamy question, the dates of the marriages are significant. For questions as to which law governs or which last will and testament prevails, dates are important. Every word, phrase, and sentence may be significant depending on the ‘eall of the question’ After reading the fact pattern and encircling some key words or phrases, an average student will start writing his answer only to make erasures halfway through. Result? A bluebook that is artistically designed with lots of arrows, obliterated letters, asterisks, and all other symbols which SRD BOOSTER: DISSECT THE ESSAY! a (Know the “Call ofthe Question” ofthe Problem) show how disorganized the student was when he was writing his answer. I have seen bluebooks with “Sir, please see back page for answer to Question 4” or a huge “X” mark covering the entire page to nullify it or some torn and/or stapled pages. ‘Most professors would consider such bluebooks as difficult to check, and consequently become stingy in giving credits. Bar examiners will definitely not have the time or the patience to read poorly organized answers. I asked one examiner of a recent Bar examination how he managed to check more than * 5000 booklets with more than 20 questions per booklet within five months while working as a lawyer/partner for a mid-sized law Firm. He said that if he reads a good first sentence, neatly written and appears to be organized, he will simply skim the next sentences and give that answer maximum or close to maximum credit. He also said that if reads a bad first sentence in terms of substance or grammatical structure or both, he is inclined not to read the next sentences out of fear of a migraine of sorts! You unconsciously made an outline of key facts in the process. If there are more than two parties, or if there is more than one transaction, This is important ‘especially in questions regarding contract questions with multiple parties and/or transactions and family law questions with multiple marriages. Next, re-read the entire question very closely, to make sure you have not missed anything. Some essay problems can be divided into distinct parts. In such cases, each part is to be solved separately. If the examiner has not divided the question into parts (as commonly is the case), the best way for you to start is to try to figure out some way to divide the question into parts. If you cannot divide it into parts, you will be forced to answer it as one single question, and if you miss the point, it will be catastrophic. -T suggest that you read and answer questions sequentially as much as possible. If you find an extremely complicated question that looks like it will take a long time to answer, save it until you have disposed of the shorter problems. It does not 18 8 PERFORMANCE BOOSTERS TO CONQUER ANY LAW EXAM benefit you to answer one difficult question and fail to answer several easy ones. But before you skip the question, make sure ‘you have at least encircled the key words that you consider as, part of the “call of the question.” That way, if and when you have the time to answer this difficult question, you don't have to waste time trying to re-read the entire fact pattern. Some exams pose a seemingly trivial or transparent question. If after careful analysis, you do not find any further issues, accept the gift cheerfully. There may be a few such questions in some law school exams. Perhaps to your surprise, you may find quite a few of those straightforward questions in ‘the Bar examinations. Some examiners will list a wide gamut of facts, with more than half irrelevant to the issue at hand. Those non-relevant facts are given to test a student's attention to detail and again the ability to spot the issue. Your grade will depend on your ability to discern the principal issues and themes presented by the question, For example, in a Contracts exam, the ultimate facts may bbe as follows: X makes an offer to Y; Y makes a counteroffer; X accepts; Y fails to perform; none of these offers were in writing. Based on the fact pattern, the legal issues would be details on how the offer was made and/or accepted; the extent or manner of the breach, the applicability of the Statutes of Frauds; and the availability of remedies. Not all issues may be asked, so just answer what the professor expects you to answer. Resist the temptation to show off and write everything you know about these issues. If you do, not only will you run out of time to answer what is being asked but you will also create a bad impression on the professor. The structure of most law school exams will test a student's ability to spot the issue and relate the law or jurisprudence to the given fact pattern. It is not too difficult without time constraints, But that is never the case in any law school exam. SRD BOOSTER: DISSECT THE ESSAY! 19 (Know the “Cal ofthe Question” ofthe Problem) 3rd Booster ILLUSTRATION: What is the call of the question? Below is a fact pattern based on an actual Bar exam question. I took out the relevant words and phrases that will help me determine the call of the question. Philippe and Nancy migrated to the United Kingdom, leaving behind their 4 children, Manny, Jenny, Penny, Danny. The spouses own a duplex apartment and allowed Danny to live in one of the units. While in the United Kingdom, Philippe died. Pending the probate of the will presented before the courts of the United Kingdom, Nancy and all of Philippe’s children executed an Extrajudicial Settlement (EJS/of Philippe’s estate wherein the duplex apartment was assigned by all the children to their mother, Naney. Subsequently, Nancy sold the property to cousin George who thereafter asked Danny to sign a prepared Lease Contract so that he (Danny) and his family could continue ‘occupying the unit. Danny refused to sign the contract alleging that his parents allowed him to continue occupying the premises. If you were George's counsel, what legal steps will you take? Explain. COMMENT: The question calls for remedies (“legal steps”) available to George so that he can at least exercise rights of ownership over the duplex — presumably to collect rent under a signed Lease Contract. The remedy available will however depend as to who has the better right of ownership and/or possession as between Danny and George. I find the phrases “pending probate,” “executed EJS,” “assigned by all the children to their mother” as the important words and phrases that will lead me to the “Call of the Question.” After all, the right of George obviously emanates from the sale between Nancy and George, the EJS among the heirs, and the death of Philippe. I find the phrases “migrated to the United Kingdom,” “duplex apartment,” “cousin George” as non-relevant. The question does not ask which law is applicable so migration to the UK is of no legal consequence. The fact that George is a cousin of Nancy has no legal relevance since the sale between them is not put into issue by Danny. Danny is simply claiming rights over the apartment by virtue of some tolerance/permission from his parents. In all likelihood, these phrases were placed 20 & PERFORMANCE BOOSTERS TO CONQUER ANY LAW EXAM by the author of the question to throw off the examinee. Did they throw you off? 8rd Booster EXERCISE: In this exercise, read the problem carefully then find the words or phrases that are likely to have legal significance to the question. Kitchie Tempo was one of approximately 500 production operators at HITEC Semiconductors, Inc., an export-oriented enterprise whose business depended on orders for computer chips from overseas. She was hired as a contractual employee four years ago. Her contracts would be for a duration of five (5) ‘months at a time, usually after a one-month interval. Her re- hiring was contingent on her performance for the immediately receding contract. ‘Six months after the expiration of her last contract, Kitchie went to HITEC’s personnel department to inquire why she was not yet being recalled for another temporary contract. She was told that her performance during her last stint was “below average.” Since there was no union to represent her, Kitchie seeks your advice as a labor lawyer about her chances of getting her job back. What will your advice be? Indicate the keywords or phrases that lead to the “Call of the Question”: 4th Booster: Spot the Issue; Win Half the Battle (Some Proven Methods to Locate an Issue in a Problem) After reading the question and writing down some key facts of the problem, you are now ready to analyze for issues, determine the applicable laws and rules, and reach your conclusions, What denotes an underlying issue? ‘An issue is sharp and to the point. For example, the issue “Is D Liable?” is too broad; it is the ultimate question, certainly not an issue with which the overall problem can be attacked piecemeal. On the other hand, “Does a contract require consideration?” is too narrow to be an issue; it is a pure question of law which does not in itself bear on the present fact problem. An underlying issue must raise a question, the answer to which exists but is not obvious. ‘An underlying issue is always a mixture of law and fact, such as “Did the release of A also act as a release of B?" This question cannot be answered without selecting the appropriate legal rule and applying the appropriate legal rule to the facts of the particular problem to see whether the answer comes out one way or the other. How do you locate an underlying issue(s)? I suggest two ‘ways to do this. the most commonly it fn in a cases, the issues stand out. It will not work well unless you are very well versed in the law and in legal analysis. It will also not work well unless you are accustomed to thinking in terms of legal rules and principles, rather than in terms of facts (if you a 22 8 PERFORMANCE BOOSTERS TO CONQUER ANY LAW EXAM are scientifically or mathematically inclined, or are accustomed to abstract logic, this may be natural to you; otherwise, it may be ineffective to attempt to locate issues in this way). Even ‘the most adept person is likely to miss some vital issues using this method. There is an almost irresistible impulse to find one issue which looks like *the” issue, when in fact there may be four or five issues requiring discussion. ‘The second way is the systematic method. This method is almost foolproof, but requires time and practice. In a question involving procedure or remedies, it involves a full-scale inves- tigation of most of the following questions: * Is each and every element of a cause of action present? * Is the defendant a proper party against whom the action can be brought? © Isthe plaintiff'a proper party to bring the action? ‘+ Does the court have jurisdiction over the subject matter and (if necessary) over the parties? * Does the defendant have an affirmative defense? * Can the issue so determined be raised at this particular stage of the proceedings? In the case of questions in procedural law as opposed to substantive law, the list can in some cases be condensed, since it may not exactly be a question of whether or not a “cause of action” exists, but whether, for example, certain evidence is admissible under the rules of law. This method should always bbe used as an analytical tool when the keyword method fails to turn up issues. It is also especially advantageous in certain areas of substantive law in which there are “logical networks” of law, eg., Contracts, Sales, Negotiable Instruments, Torts, and Criminal Law. It is much less valuable in Constitutional Law and in other subjects, such as Corporation Law, in which formal “causes of action” are not exactly involved. In most cases, the use of the systematic method will lead to an accurate recognition of issues and, hence, a higher score on the question. Remember, half the battle is won when you spot the right issue(s)! ‘4TH BOOSTER: SPOT THE ISSUE; WITH HALF THE BATTLE 23 (Gome Proven Methods to Locate an Issue in a Problem) Issue spotting is not rocket science. As long as you read and understand the problem (as in knowing the ‘call of the question”) and encircle the key words or phrases, your basic knowledge of the law will lead you to the underlying issue. Take this example. “On Dec 1, 1990, X was waiting in a bus stop in Makati that was coincidentally in front of a bank. Y, rookie police officer, accosted X. ¥ noted that X was acting suspiciously since he was wearing a cap and carrying a backpack. Without asking any questions, Y immediately stopped and frisked X and discovered marijuana sticks inside his wallet. X was arrested without a warrant, questioned without the assistance of legal counsel, and charged with possession of dangerous drugs. Was the arrest lawful?” What are the words that are relevant to the question ‘was the arrest lawful?’ Since the legality of the arrest depends on probable cause, then the key phrase in that question is “acting suspiciously.” Such phrase will now help you determine if there was probable cause. Additionally, the date plus the fact that the bus stop was near a bank, (period where there were several armed robberies in 1990) and the term “rookie” (ability to discern suspicious activities by experience) would be relevant in your determination whether there was probable cause. 4th Booster EXERCISE: For the actual Bar exam questions listed below, choose among the given answer(s) which best spots the issue(s). Atty. Emily Go, a legitimate daughter of a Chinese father and a Filipino mother, was born in 1945. At 21, she elected Philippine citizenship and studied law. She passed the bar examinations and engaged in private practice for many years, The Judicial and Bar Council nominated her as a candidate for the position of Associate Justice of the Supreme Court. But her nomination is being contested by Atty Juris Castillo, also an aspirant to the position. She claims that Atty. Emily Go is not a natural-born citizen, hence, not qualified to be appointed to the ‘Supreme Court. Is this contention correct? 1. Whether the fact that Atty. Go passed the Bar and practiced law for several years made her a natural- born citizen. 24 8 PERFORMANCE BOOSTERS TO CONQUER ANY LAW EXAM 2. Whether mere election of Philippine citizenship at the age of 21 made Atty. Go a natural born citizen of the Philippines. 3. Whether being a legitimate daughter of a Chinese father and a Filipino mother allowed Atty. Go to elect Philippine citizenship. 4. Whether the fact that Atty. Go was born in 1945 allowed her to elect Philippine citizenship. 5. Whether Atty. Castillo has the personality to challenge the citizenship of Atty. Go. 6. Whether the issue of being a natural born citizen can no longer be contested since the Judicial and Bar Council nominated Atty. Go for the position of Associate Justice of the Supreme Court. 7. Whether Justices of the Supreme Court must be natural-born Filipino citizens. Atty. Richard Chua was born in 1964. He is a legitimate son of a Chinese father and a Filipino mother. His father became a naturalized Filipino citizen when Atty. Chua was still a minor. Eventually, he studied law and was allowed by the Supreme Court to take the bar examinations, subject to his submission to the Supreme Court proof of his Philippine citizenship. Although he never complied with such requirement, Atty. Chua ‘practiced law for many years until one Noel Eugenio filed with the Supreme Court a complaint for disbarment against him on the ground that he is not a Filipino citizen. He then filed with the Bureau of Immigration an affidavit electing Philippine citizenship. Noel contested it claiming it was filed many years after Atty. Chua reached the age of majority. Will Atty. Chua be disbarred? Explain. 1. Whether election of Philippines citizenship can be made anytime. 2. Whether Atty. Chua can be disbarred for failing to elect Philippine citizenship. 3. Whether the Supreme Court can disbar Atty. Chua due to his failure to elect Philippine citizenship at the time he took the Bar examination. 4TH BOOSTER: SPOT THE ISSUE; WITH HALF THE BATTLE 25 (Gome Proven Methods to Locate an Issue in a Problem) 4. Whether Atty. Chua should be disbarred since he was ino citizen at the time of taking of the Bar examination. Whether Atty. Chua should be disbarred since he was not a Filipino citizen at the time of filing of the disbarment case and whether that the fact of election of Philippine citizenship AFTER the complaint was filed is proper. 6. Whether Atty. Chua is guilty of fraud for taking the Bar exam without disclosing the fact of his citizenship before the Supreme Court. 7. Whether Atty. Chua's failure to submit proof of his citizenship is a ground for disbarment, ‘4TH BOOSTER: SPOT THE ISSUE; WITH HALF THE BATTLE 25 (Gome Proven Methods to Locate an Issue in a Problem) 4. Whether Atty. Chua should be disbarred since he was not a Filipino citizen at the time of taking of the Bar examination. 5. Whether Atty. Chua should be disbarred since he was not a Filipino citizen at the time of filing of the disbarment case and whether that the fact of election of Philippine citizenship AFTER the complaint was filed is proper. Whether Atty. Chua is guilty of fraud for taking the Bar exam without disclosing the fact of his citizenship before the Supreme Court. 7. Whether Atty. Chua's failure to submit proof of his citizenship is a ground for disbarment. 5th Booster: Make a Road Map. (An outline organizes your answer. Translation: Maximum Points) Take note of my suggested time allocation. The time to write down the answer is only half the time allotted to make an outline and to understanding the problem. It only goes to show the importance of having a road map before going somewhere. You may have heard of “IRAC” which stands for Issue, Ruling, Analysis, and Conclusion. That formula works well in most hypothetical exam questions. With IRAC, your answer can be as short as 4 sentences. In U.S. law school exams, professors discourage one-sided arguments, meaning your answer should also present the other side. For Philippine law school exams, we are trained to be direct to the point. Either way, the most. important exercise is to spot the issue and resolve it with legal bases. SAMPLE QUESTION: ROGER CRUZ is accused of ‘MURDER in an Information quoted below: “That on or about 11:00 p.m. of August 16, 2003 at Fordham Street, Quezon City and within the jurisdiction of the Honorable Court, ROGER CRUZ, with intent to kill, unlawfully and feloniously waited for his friend JUAN SANTOS in the dark alley near the latter’s house and when JUAN SANTOS reached the darkest portion of the ‘alley, ROGER CRUZ clubbed him to death with a golf club who died instantly as he was hit on the head several times.” ROGER was arrested by a Barangay Tanod who saw him about to leave the crime scene and found the golf club in his possession. He was brought to the police station and was detained for two weeks. Inquest investigation was conducted 26 STH BOOSTER: MAKE A ROAD MAP 2 (An Outline organizes your answer. Translation: Maximum Points) and the Information as quoted above was filed in the Regional ‘Trial Court of Quezon City. Was the arrest of ROGER by a Barangay Tanod legal? What if he was arrested by a private person, will the arrest be legal? Explain (15%) SUGGESTED ANSWER: Yes, the arrest of ROGER by a Barangay Tanod is legal. Under Sec. 5, Rule 113 of the Revised Rules in Criminal Procedure, an arrest without warrant would still be lawful when “an offense has just been committed and he has probable cause to believe based on personal knowledge of facts or circumstances that the person to be arrested has ‘committed it.” In the problem, it was clear that ROGER was ‘caught about to leave the crime scene and the Barangay Tanod has probable cause to believe based on personal knowledge of the circumstance that ROGER has committed it since he was caught with the murder weapon. If the arrest was made by a private person, I will maintain my answer, the provision of the law speaks of a peace officer or a private person who may arrest a person under the circumstance just stated. Focusing on the key words/phrases will ultimately lead you to the issue, Having spotted the issue, we now proceed to make an outline. Refer to our sample problem on arrest without warrant. Your outline should be structured like this: © Issue(s) — Forget the ultimate issue (valid or invalid arrest without a warrant), focus on the underlying issue (was there probable cause in the facts given?) ‘© Ruling or Law — Recall applicable jurisprudence and Bill of Rights on arrests without warrant * Analysis — What constitutes “acting suspiciously” that will give rise to probable cause? Focus on those key words or phrases in the question. (Xwearingacap and carrying a backpack waiting in front of a bank, a rookie police officer, no probing questions, 1990 where a lot of several bank robberies occurred). i ‘Sapien — reiterate your straight answer and lude provisos and/or exceptions if applicable. 28 8 PERFORMANCE BOOSTERS TO CONQUER ANY LAW EXAM Cases involving arrests without warrants can goeither way depending on the appreciation of the facts and circumstances at hand. Having an outline will insure that you cover such key words or phrases before writing down your answer. An outline will not only minimize erasures but will also help you organize your answer. Ifour sample question was asked in a U.S. law school exam, your answers would have to be organized slightly differently. The professor would expect you to give him countervailing arguments, meaning you have to present the arguments of bboth sides before choosing one over the other. Similar to most of our Supreme Court decisions, your answer has to present both sides of the argument with their respective legal bases, if any, then explain why the other side makes better legal sense over the other. Some students can make the IRAC outline mentally. No need to write an outline since it is a waste of time they say. But what if you had to skip the problem since the answer could go either way? What if halfway through writing your answer, you remembered a case that will change your position/answer? Result? Exhibit 1: a Picasso-like bluebook! Once you have allotted the time for each question, you will always have the time to write an outline. Remember that ‘the substance of your answer serves as the basis for your grade. A law exam is not a writing contest where the longest answer gets the best grade. Lawyers are not paid to write, they are paid to think. Law students are supposed to think and analyze before writing down their answers.(Underscoring supplied for emphasis) 5th Booster ILLUSTRATION: The Making of a Road Map: Bar Question: Saul, a married man, had an adulterous relation with Tessie. In one of the trysts, Saul’s wife, Cecile, ‘caught them in flagrante. Armed with a gun, Cecile shot Saul in 4@ fit of extreme jealousy, nearly killing him. Four (4) years after the incident, Saul filed an action for legal separation against Cecile on the ground that she attempted to kill him. ‘STH BOOSTER: MAKE A ROAD MAP ~ (An Outline organizes your answer. Translation: Maximum Points) 1. If you were Saul’s counsel, how will you argue his case? 2. If you were the lawyer of Cecile, what will be your defense? Keywords leading to the Underlying Issue: “shot Saul, Road Map for an Answer to Question 1: + Issue(s) — Whether Cecile’s act of attempting to kill Saul “in a fit of jealousy” is a ground for legal separation and whether the action has prescribed due to the lapse of four years. + Ruling or Law — An attempt on the life of the petitioner is a valid ground under Article 55 of the Family Code. Legal separation may be filed within five (5) years from the time of the occurrence of the same. © Analysis — + Is an evil design to kill the spouse necessary to constitute a ground for legal separation? The Code did not distinguish whether such attempt was intentional, by negligence, or attended by some kind of justifying circumstances, © Does the lapse of 4 years constitute “condonation” or waiver of the prescriptive period of 5 years? * Conclusion — Road Map for an Answer to Question 2: ‘* Issue(s) — Whether the attempt to kill Saul “in a fit of jealousy” constitutes a valid ground for legal separation. Whether the concept of in pare delicto applies in this situation. + Ruling or Law The avowed purpose of Article 55 of the Family Code in granting petitioner the ground to file legal separation is to punish or penalize the respondent for some kind of “misdeed” or fault. If ‘5TH BOOSTER: MAKE A ROAD MAP a (An Outline organizes your answer. Translation: Maximum Points) As counsel of plaintiff Baker, argue why his complaint should not be dismissed on the ground of defendant Adams’ diplomatic immunity from suit. As counsel of defendant Adams, argue for the dismissal of the complaint. DRAFT AN OUTLINE USING THE IRAC METHOD Keywords leading to the Underlying Issue in the set of questions: As counsel of plaintiff Baker, argue why his complaint should not be dismissed on the ground of defendant Adams’ diplomatic immunity from suit. © Issue(s) — * Ruling or Law — © Analysis — © Conclusion — ‘8 PERFORMANCE BOOSTERS TO CONQUER ANY LAW EXAM As counsel of defendant Adams, argue for the dismissal of the complaint. © Issue(s) — © Ruling or Law — © Analysis — * Conclusion — Network to host a one-hour daily talk show where she interviews various celebrities on topical subjects that she herself selects. ‘5TH BOOSTER: MAKE A ROAD MAP 3 (An Outline organizes your answer. Translation: Maximum Points) She was paid a monthly remuneration of P300,000.00. The program had been airing for almost two years when sponsors’ advertising revenues dwindled, constraining the network to cancel the show upon the expiration of its latest contract with ‘Ms. Vartan. The talk-show host protested the discontinuance of her monthly talent fee, claiming that it was tantamount to her illegal dismissal from the network since she has already attained the status of a regular employee. As the network's legal counsel, how would you justify its decision to cancel Ms. Vartan's program which in effect terminated her services in the process? ‘As counsel for the talk-show host, how would you argue your case? DRAFT AN OUTLINE USING THE IRAC METHOD, Keywords leading to the Underlying Issue in the set of questions: ‘As the network’s legal counsel, how would you justify its decision to cancel Ms. Vartan’s program which in effect terminated her services in the process? © Issue(s) — + Ruling or Law — 34 8 PERFORMANCE BOOSTERS T0 CONQUER ANY LAW EXAM © Analysis — ‘As counsel for the talk-show host, how would you argue your case? * Issue(s) — ‘STH BOOSTER: MAKE A ROAD MAP 35 (An Outline organizes your answer. Translation: Maximum Points) © Conclusion — 6th Booster: Write a Masterpiece! (Excellent composition, easier to check, more credits) It is vital, after completion of the legal analysis, to make sure that you have answered precisely the question asked, completely answered it, and stated your conclusion as to who wins what against whom in an unambiguous language. Respond to the issue of the question squarely: Yes or No; Guilty or Not Guilty; Void, Voidable, or Valid. Not only will it be easier for the examiner to check, but a categorical statement reflects a high degree of confidence in your answer. I have seen answers that started out with: “No less than Section 2 of Article III of the 1987 Constitution provides that no unreasonable searches or seizures ....” and something like: “Since time immemorial, ‘meeting of the minds is the essence of any contract...” Cut to the chase; be direct to the point. For answers like that can go either way, I strongly discourage starting your answer by “it depends” Choose one side, then use conditional or exception language such as, “Yes, as long as...” or “No, provided that .. Again, beginning your answer with a definite statement of your position shows a high degree of confidence in your answer. Be conscious of your grammar, conjugation and spelling. 1 offer no cure for English deficiency since it is assumed that one who is admitted to law school has a good command of English and can express himself in the language both verbally and in writing. I once saw an answer that looked like this: “X seeked an injunction to protect his interest.” How can I appreciate an answer with such a glaring grammatical error? The same goes with spelling, I have encountered quite a few freshmen who spelled marriage as “marraige” and receive as “recieve.” Quite amusing for the professor but a deduction for a misspelled word or wrong grammar is never amusing for a law student. For lawyers (and would be lawyers) proficiency in the English language is indispensable. 6TH BOOSTER: WRITE A MASTERPIECE! 7 (Excellent composition, easier to check, more credits) Write neatly. Effectivetimemanagement and an organized outline will almost always lead to a neat examination booklet. I once had a complaint from a student who said that his answers were very similar to those of his classmate but he received a lower grade. When I reviewed their examination booklets, he had some basis. I could only surmise that I probably gave his classmate a higher grade due to handwriting differences. The complaining student's handwriting was a bunch of disorderly and garbled letters, whereas his classmate’s handwriting was impeccably neat which made his answers stand out and easier to check. Consider using print instead of script if the former style of writing will be easier to read. I had a classmate whose handwriting was script throughout his 4 years in law school. Having been cautioned by more than one professor, he decided to use print for the Bar examinations. He nearly made it to the ‘Top Ten that year. Develop a good legal style of writing. Write “do not” instead of “don’t.” Avoid using acronyms such as SC for the ‘Supreme Court or NCC for the New Civil Code. The use of these acronyms might save you time but it makes your written work unprofessional or less lawyerly. edeneiahlaeenipavins, 1 ioe that nit of stadt are captivated wit court opinions are written with Jong sentences and a considerable amount of legal jargon. As law students, you are not expected to write these fancy words unless it is imperative to use them. ins. Have you seen a good painting lately? Inanart gallery, you will almost always find paintings that are mounted on frames with borders. Most art critics appreciate the work if, at first glance, itis refreshing to the eye. Margins will have that same effect in your answers. Do everything to make it easier for the examiner to check your exam booklet. Believe me, it makes a world of difference if your answers are presented in a manner that is easier to check. er. Unless an answer concludes with an unequivocal statement of who wins what against whom, it is incomplete. This unequivocal statement must be included even 98 8 PERFORMANCE BOOSTERS T0 CONQUER ANY LAW EXAM when the examiner asks for a “discussion” or asks a question such as “What defenses can D raise?” One must also include the line of reasoning used to arrive at the answer, even if the examiner has not expressly asked for anything other than “Who wins what?” ‘You will maximize your scores when you make sure that ‘your answer is precise, readable, and complete! 6th Booster EXERCISE: With a well-written work, you actually help the checker give you a better grade since your answers are easier to understand. In this exercise, give ‘yourself ten minutes to answer the following question. Restrict your answer to the space provided or one page of a standard blue book. QUESTION: Philippe and Nancy migrated to the United Kingdom, leaving behind their 4children, Manny, Jenny, Penny, Danny, The spouses own a duplex apartment and allowed Danny tolive in one of the units. While in the United Kingdom, Philippe died, Pending the probate of the will presented before the courts of the United Kingdom, Nancy and all of Philippe’s children executed an Extrajudicial Settlement of Philippe’s estate wherein the duplex apartment was assigned by all the children to their mother, Nancy. Subsequently, Nancy sold the property to George who thereafter asked Danny to sign a prepared Lease Contract so that he (Danny) and his family could continue occupying the unit. Danny refused to sign the contract alleging that his parents allowed him to continue occupying the premise If you were George's counsel, what legal steps will you take? Explain, 6TH BOOSTER: WRITE A MASTERPIECE! (Bxcellent composition, easier to check, more credits) Below are ten ACTUAL answers (with comments) to the same question given during a 10 minute quiz in a major law school in Metro Manila. SAMPLE ANSWER 1: “If I were George's counsel, I would sue for specific performance to enforce the contract of sale between George and Nancy. Upon the death of Philippe, the rights and obligation were immediately transferred by operation of law to his heirs — Naney and his 4 children. When the heirs extrajudicially settled the estate of Philippe wherein the duplex ‘apartment was assigned to Naney, in effect, there was already a partition of the estate. Therefore, when Nancy sold the property to George, Nancy, as assignee to the duplex apartment, had the right to dispose of the property; hence, she validly transferred the property to George. And since Danny refuses to sign the Lease Contract, George can file a suit for specific performance. Also, if Danny still refuses to sign, he can file a suit for ejectment since Danny doesn’t have a right to occupy the apartment, the right to ‘occupy belonging to the owner, George.” COMMENTS: Wrong answer, partially right analysis. The remedy of specific performance cannot be ‘used as against Danny who was not privy to the sale between George and Nancy. The answer is correct in saying that upon death, rights and obligations (property was inadvertently omitted perhaps) are transmitted to the heirs under Article 777 of the Civil Code. The answer however missed the issue of whether or not heirs can dispose of property pending probate of the will. The last sentence appears to be a last minute insertion; obviously 40 8 PERFORMANCE BOOSTERS T0 CONQUER ANY LAW EXAM indicating the lack of an outline and a failure to spot the issue. SAMPLE ANSWER 2: “As the counsel of George, the first step necessary would be to determine the validity of Philippe's will as regards his Testamentary Capacity and the Formalities required by law. Pending the probate of the will in the UK, Imust also determine if the laws of the UK (since Philippe and Nancy migrated, they are governed by the UK laws or Settlement of Estate) and the laws of the Philippines coincide, in accordance with the Renvoi Doctrine of the Conflict of laws. That is when I can begin to ask the Court regarding the nature of the transfer made by Nancy (sale) as the assignee of all the heirs of Philippe, to George, as third person purchase for value. George must prove that he is an innocent purchase for value so that his right to the property would not be impaired by future probate proceeding and the adjudication of the final settlement of the estate. As the counsel, I must prove that Extrajudicial Settlement is valid by virtue of the portion alienated by Nancy, with respect to her share in the conjugal assets and the validity of Lease Contract between George and Danny. Danny may not refuse to sign the Lease Contract upon the finding of the Court that George as an innocent transferor has a better right, assuming that all the presumptive legitimes of the compulsory heirs will not be impaired by such transfer.” COMMENTS: Hodgepodge and a show-off! The answer is a laundry list ofall ofthe possible steps without considering the “call of the question”. The question did not ask for the application of conflicts rules, the concept of “innocent purchaser and for value”, and the delivery of presumptive legitimes. The examinee evidently wrote everything what he can in ten minutes without analyzing. the fact pattern. This answer is regretfully disastrous especially in a Bar exam where examinees are expected to zero in the issue presented by the fact pattern. The last two sentences are more attuned to the issue as to whether an heir can dispose of assets of the decedent pending the probate of the will. SAMPLE ANSWER C: “Succession is a mode of acquisition whereby the property, rights and obligations of a person is 6TH BOOSTER: WRITE A MASTERPIECE! a (Excellent composition, easier to check, more credits) transmitted through the heirs of the decedent. As a counsel of George, I will first establish that the heirs are now the owners of the duplex apartment. Having done that, I will then establish ‘that as a co-owner of the property, he (Danny) assigned his rights over the same by virtue of the extrajudicial settlement executed over Philippe's estate. With the valid extrajudicial settlement Nancy may then exercise dominion over said property and may alienate the same to a lawful person under the law. Since there is no question as to the validity of sale, Danny who already assigned his rights to Nancy, may not occupy the property without beneficial ownership or lawful title. Thus, George is now the one with rights over the property under the law.” COMMENTS: Prose or poetry? It is not wise to start, your answer with a statement that is not responsive to the question. The checker obviously knows the definition of succession and there is no need to write the same in this ten minute quiz! What is the legal basis to establish that the heirs become co-owners of the duplex apartment? This is the heart of the question but the answer failed to cite legal basis, which is Article 777 of the Civil Code. The answer contains sweeping conclusions as it considered the sale valid because the extrajudicial settlement is valid. ‘The answer is also flawed logically. ‘SAMPLE ANSWER D: “Mr. George has the legal right to evict Danny from the premise. By virtue of extrajudicial settlement executed between the heirs, Nancy was given the absolute ownership over the duplex apartment, Her subsequent transfer of the said property to Mr. George is valid. There was no showing that the assignment made by the heirs in favor of Naney was tainted by any fraud which would invalidate the transfer.” COMMENTS: Move to Strike for not being responsive to the question, Your Honor!” The question asked for the “steps” not whether George has the “right”. Ifthe question is stated as “Does George have a legal right over the property?,” then the answer is responsive. The answer is also flawed logically as it assumes that the extrajudicial settlement executed among the heirs is valid. It also failed 40 8 PERFORMANCE BOOSTERS TO CONQUER ANY LAW EXAM indicating the lack of an outline and a failure to spot the issue. SAMPLE ANSWER 2: “As the counsel of George, the first step necessary would be to determine the validity of Philippe’s will as regards his Testamentary Capacity and the Formalities required by law. Pending the probate of the will in the UK, Imust also determine if the laws of the UK (since Philippe and Nancy migrated, they are governed by the UK laws or Settlement of Estate) and the laws of the Philippines coincide, in accordance with the Renvoi Doctrine of the Conflict of laws. That is when I ‘can begin to ask the Court regarding the nature of the transfer made by Naney (sale) as the assignee of all the heirs of Philippe, to George, as third person purchase for value. George must prove that he is an innocent purchase for value 80 that his right to the property would not be impaired by future probate proceeding and the adjudication of the final settlement of the estate. As the counsel, I must prove that Extrajudicial Settlement is valid by virtue of the portion alienated by Nancy, with respect to her share in the conjugal assets and the validity of Lease Contract between George and Danny. Danny may not refuse to sign the Lease Contract upon the finding of the Court that George as «an innocent transferor has a better right, assuming that all the presumptive legitimes of the compulsory heirs will not be impaired by such transfer.” COMMENTS: Hodgepodge and a show-off! The answer is a laundry list of all of the possible steps without considering the “call of the question’. The question did not ask for the application of conflicts rules, the concept of “innocent purchaser and for value”, and the delivery of presumptive legitimes. The examinee evidently wrote everything what he ean in ten minutes without analyzing the’fact pattern. This answer is regretfully disastrous especially in a Bar exam where examinees are expected to zero in the issue presented by the fact pattern. The last two sentences are more attuned to the issue as to whether an heir can dispose of assets of the decedent pending the probate of the will. SAMPLE ANSWER C: “Succession is a mode of acquisition whereby the property, rights and obligations of a person is 6TH BOOSTER: WRITE A MASTERPIECE! a (Excellent composition, easier to check, more credits) transmitted through the heirs of the decedent. As a counsel of George, I will first establish that the heirs are now the owners of the duplex apartment. Having done that, I will then establish that as a.co-owner of the property, he (Danny) assigned his rights ‘over the same by virtue of the extrajudicial settlement executed ‘over Philippe’s estate. With the valid extrajudicial settlement ‘Naney may then exercise dominion over said property and may alienate the same to a lawful person under the law. Since there is no question as to the validity of sale, Danny who already assigned his rights to Nancy, may not occupy the property without beneficial ownership or lawful title. Thus, George is now the one with rights over the property under the law.” COMMENTS: Prose or poetry? It is not wise to start your answer with a statement that is not responsive to the question. The checker obviously knows the definition of succession and there is no need to write the same in this ten minute quiz! What is the legal basis to establish ‘that the heirs become co-owners of the duplex apartment? ‘This is the heart of the question but the answer failed to cite legal basis, which is Article 777 of the Civil Code. The answer contains sweeping conclusions as it considered the sale valid because the extrajudicial settlement is valid. ‘The answer is also flawed logically. SAMPLE ANSWER D: “Mr. George has the legal right to evict Danny from the premise. By virtue of extrajudicial settlement executed between the heirs, Nancy was given the absolute ownership over the duplex apartment, Her subsequent transfer of the said property to Mr. George is valid. There was no showing that the assignment made by the heirs in favor of Naney was tainted by any fraud which would invalidate the transfer.” COMMENTS: Move to Strike for not being responsive to the question, Your Honor!” The question asked for the “steps” not whether George has the “right”. If the question is stated as “Does George have a legal right over the property?,” then the answer is responsive. The answer is also flawed logically as it assumes that the extrajudicial settlement executed among the heirs is valid. It also failed 42 8 PERFORMANCE BOOSTERS TO CONQUER ANY LAW EXAM to address the issue of a pending probate proceeding in the UK. SAMPLE ANSWER E: “If I were George’s counsel I will file a case of unlawful detainer against Danny for refusing to vacate the premises. In this case, the right of Danny to inherit his father’s properties accrued at the moment of death of the latter under Article 777 of the Civil Code. From this time, Danny had @ right to succeed. However, also upon the moment of death of Philippe, the duplex occupied by Danny was converted into a co- ownership. Upon the execution of the Extrajudicial Settlement, Danny lost his right to the duplex apartment whereby he assigned his rights thereto to his mother. Nancy now had full Tights over the apartment which she eventually sold to George. Danny, therefore, had no more right to occupy the duplex apartment and is unlawfully detaining the property.” COMMENTS: Precise, concise, and to the point. Except for some deficiencies in choice of words, this answer ‘managed to understand the call of the question and spot the issue. It could have obtained maximum credit had the answer addressed the key phrase of “pending probate.” SAMPLE ANSWER F: "I.would fileacivil case foruntawful detainer against Danny. Danny no longer has a right to the use of a unit in the duplex apartment, because the extrajudicial settlement in which he took part assigned the apartment to Nancy, who thereafter sold it to George. Danny does not contest the validity of these said contracts, such as that there is a will and there are debts as to the settlement, and there is lack of consideration as to the lease. Thus, Danny's consent validated the extrajudicial settlement, which requires a lack of will.” COMMENTS: A mortal sin! The biggest mistake you can commit is to introduce facts or make assumptions ‘outside of what is given in the fact pattern. The fact pattern of the question obviously did not inelude the presence of a “will,” “debts of the estate,” or “consideration of the lease.” As such, these are not essential to the issue raised by the question, (6TH BOOSTER: WRITE A MASTERPIECE! a (Excellent composition, easier to check, more credits) SAMPLE ANSWER G: “If I were George's counsel, I will secure the rights of my client by telling Danny that since there was an Extrajudicial Settlement of Philippe's estate and that the property sold by Nancy was already solely hers at the time of the sale, then George as full owner has legal right to ask Danny to sign the Lease Contract. Second, I would go to the Register of Deeds armed with the said Extrajudicial Settlement of Philippe’s estate as well as the contract of sale between my client George and Nancy 80 that a Transfer Certificate of Title covering the subject duplex apartment be placed under the name of George, who is the legal owner of the property. Third, Twill file a case Danny for illegally remaining in the George's property without a valid lease contract as George is already the ‘owner of the apartment by virtue of the sale between Nancy and George.” COMMENTS: A practical answer but not a legal answer! The answer fails to crystallize the underlyingissue of who has the better right of possession as between George and Danny. A certificate of sale is not even necessary to establish your right of ownership or possession. The steps as listed in the answer may arguably be correct; however legal basis must be given. Since the answer failed to recite those legal concepts that will address the disposition of a property pending probate (extrajudicial settlement by the co-heirs), then the answer does not merit sufficient credit. SAMPLE ANSWER H: ‘If I were George’s counsel I would assert that George has a right to the subject property by first securing the title of such property from Nancy, the seller. Even though the probate of the will before the UK court is still pending, the heirs of Philippe already executed an Extrajudicial Settlement of Philippe's estate. Pursuant to such, all the children assigned the property to their mother. An assignment is a sale and in this case, the assignment ‘made between the children and the mother is valid because the inheritance is already determinable at that point. By virtue of the laws in succession, all properties, rights and obligation of the decedent are transferred from the moment of death. Hence, 44 8PERFORMANCE BOOSTERS TO CONQUER ANY LAW EXAM by virtue of such, the heirs already acquired their portion of the ‘estate of the decedent and has assigned co-heir Nancy, who now becomes the owner of the whole property. Naney, being the owner can validly sell it to George. And by virtue of such sale, George has acquired ownership of such ‘property and hence, can properly demand that Danny execute a contract of lease should the latter decide to continue to stay on the said property.” COMMENTS: Excellent analysis but below average conclusion! The answer obviously managed to spot the issue and cite the legal basis for its arguments. However, the response of “securing the title of such property” and “properly demand that Danny execute a contract of lease” do not really fall under the category of “legal steps.” SAMPLE ANSWER I: “I would first check the citizenship of Philippe in order to identify which laws are applicable. In this case, since Philippe died in England, there might be different set of Rules regarding Extrajudicial Settlement. After that, I would check the validity of extra judicial partition with respect to the properties in the Philippines. If proven to be valid, would transfer the title to Mr. George and have it recorded in the Registry of Deed. Since the title is already in my (illegible) I would send a notice to Mr. Danny to vacate as soon as possible. If still refuses to leave, I would send a final demand letter in preparation for a case for unlawful detainer. Since Danny already sign the extrajudicial partition (blank spaces).” COMMENTS: Finishing badly! The answer is incomplete indicating poor time management. The issue was not addressed squarely indicating failure to know the call of the question. Plus, the action word “check” is not exactly a good action word for a “counsel.” SAMPLE ANSWER J: “If I were George's counsel, I will file an action for ejectment against Danny. I will allege that Danny is unlawfully detaining one of the units in the duplex I legally bought from Nancy. ‘My theory would be that upon Philippe’s death, by virtue of Article 777 of the Civil Code, Philippe’s inchoate right over the (6TH BOOSTER: WRITE A MASTERPIECE! “5 (Excellent composition, easier to check, mare credits) duplex (since presumably he co-owns it with his wife, Nancy) transferred to Nancy and all their children, being compulsory heirs of Philippe. Absent a judicial declaration ofa probate, the extrajudicial settlement — where in effect, Nancy completely owns the duplex ‘apartment —is valid and binding upon the parties. Hence, Nancy being the complete owner of the duplex apartment, legally and validly sold the duplex to my client, George. George, being the buyer is deemed to be the complete owner of the apartment. Hence, he may institute the action.” addressed the issue squarely and cited legal basis. If reduced/organized to 4 sentences, this answer could have gotten the highest grade in the quiz. 7th Booster: Know the Enemy! (Having a crystal bow! won’t hurt) ‘There was one student who spent much effort getting hold of my previous examinations. He read and answered these previous examinations. Although he also probably read the textbook, he heavily relied on these past exams. For that particular exam that he took, I made a new set of questions. I leave it up to you to guess what grade he received. Knowing your professor would be helpful in trying to understand his views and inclination to a particular issue. It also helps to know his tendency to focus on certain topics for his examinations. Even for the Bar examinations, examinees exert effort in trying to determine the identity of the examiner(s) for such exam(s), in order to get an idea albeit vague about the possible questions to be asked. Once identified, examinees or the schools’ respective bar operations committees do their investigation. How do you proceed to investigate? There are three suggested detective activities that you need to do to ‘detect’ what the questions your professor will probably ask in his exam. First, ascertain your professor’s interests. If your Constitutional Law professor is writing a book on the rights of the.accused, he will most likely ask a few questions on this topic. If your Criminal Law professor is a litigator and has handled several homicide cases, he will likely throw in some homicide questions in his exam. In one of my exams, I actually patterned my questions after an actual estate settlement case that my law firm had handled. Second, take note of the time spent by your professor on a particular topic. During the course of the semester, you will 46 ‘7TH BOOSTER: KNOW THE ENEMY! a (Having a erystal bow! won't hurt) notice that your professor will allot more time to certain topics, typically the significant ones. Conversely, he may skim other topies which he thinks are not that important or which he has little interest in. In all likelihood, he will formulate most of his questions on the topics he discussed. Of course, one can encounter some eccentric professors who will throw a curveball from time to time just to keep his students guessing. ‘Third, read your professor's past exams. They will indicate his inclination towards certain topics and positions on key issues. Though some professors can be more innovative than others, there is a finite number of topics in any given subject. Past exams are excellent guides. However, do not rely solely on these past exams for your exam preparation. As my student found out, itis always possible that a professor will write anew set of questions totally different from his previous exams. Even when you have an idea of the possible questions, do ‘not lose sight of the big picture. Too mitch fixation on detail will have its ill effects. ‘th Booster TALK: In the Philippine Military Academy, cadets are subjected not only to a rigorous physical training but also to regular written exams and surprise pop quizzes. Due to the daily physical exercises during the day, cadets find it difficult to stay awake in class and consequently know what will be covered in the written exam. In an effort to keep his students awake and make them pay attention to his lectures, a well-respected military instructor’ in PMA once said to his students - “if I will say it once, deem it as nice to know; if I will say it twice, consider it as important to know; and if I say it three times, it will be part of your exams!” "It. Gen, (Ret) Salvador Mison taught in the Philippine Military Acad- ‘my from 1962 to 1968, He was the Vice Chief of Staff, AFP under President Corazon C Aquino and, upon his retirement from the AFP in 1987, he was ap- pointed Commissioner ofthe Bureau of Customs from 1987 to 1991. 8th Booster: Conquer All Others! (True/False, Multiple Choice, the Bar Exams) There will be rare occasions when you will be given an ‘open book or even a take home exam. Some 2nd or 3rd year level professors, especially those teaching elective courses, may give you these kinds of exams. I suggest you use the same approach as you would take in a closed book exam. Do not be complacent. The time pressure will always be there even in open book exams. Trecommend that you prepare a checklisoutline that will help you extract legal terms used in the course. For instance, the Revised Penal Code provides the definition of coup d'etat as follows: “Article 134-A. Coup d'etat; How committed. — The crime of coup detat is a swift attack accompanied by violence, intimidation, threat, strategy or stealth, directed against duly constituted authorities of the Republic of the Philippines, or any military camp or installation, communications network, public utilities or other facilities needed for the exercise and continued possession of power, singly or simultaneously carried out anywhere in the Philippines by any person or persons, belonging to the ilitary or police or holding any public office ofemployment with or without civilian support or participation for the purpose of seizing or diminishing state power.” For an open book exam, an average student will look for the page(s) that refers to coup d’etat. He may have indexed his Code or even marked the key elements with a highlighter in his Code. An above average student will make a short version of these elements of rebellion in an outline that may look like: “swift attack, violence or intimidation, against RP or vital facilities, by military or police, purpose is to seize state power.” 6 STH BOOSTER: CONQUER ALL OTHERS! co (True/False, Multiple Choice, the Bar Exams) ‘A two-page outline containing all the key elements of ‘major crimes in the Revised Penal Code will be easier to refer to compared to a 30-page Code or worse a 300-page textbook annotation. You do not have to outline each felony, just focus con the ones discussed in class or emphasized by the professor. Thave never given an open book exam as a professor but I have taken one as a student. I remembered making outlines not only to serve as guides but also to help me memorize those key ‘words I need. On these exams, I referred to my textbook only to review what I wrote. I looked at my outlines a few times, but for the most part I wrote my answer using the same approach allotted the time, made an outline, then wrote my answer. By ‘experience, you will run out of time if you rely on your books ‘as reference during the exam. The moment you see classmates browsing through their textbooks during an open book exam, ‘you will either panic since there will be definitely things in tthe book not in your outline or feel relieved that you made an ‘outline so that you don’t have to waste time like them. Again, panic is your worst enemy and not the lack of knowledge. If ‘you studied as if the exam was closed-book, fear not, you are ‘equipped to pass that exam. ‘There may be objective questions in some law exams. In a True or False exam, my suggestion is to trust your instinct. But you have to understand what the assertion in the question says. After reading AND understanding the question, you will instinctively have an answer. Your first impulse is more than likely correct. Take this question for example: “Traffic in Metro Manila is bad!” (It might be poorly drafted but nonetheless you have to give an answer). Instinctively, you will answer “True.” But upon deeper reflection, you can say that traffic is not bad during the weekends or at night or traffic is not bad in some parts of Metro Manila. This process has now caused you to doubt your answer causing you anxiety to the extent that you may change your answer. Go with your first impulse, Trick questions will be given sometimes but not all the time. Do not spend a lot of time on these questions; on average, these trick questions represent less than 10% of those fair and straightforward True or False questions. 508 PERFORMANCE BOOSTERS T0 CONQUER ANY LAW EXAM In a Multiple Choice exam, I recommend the same thing: trust your instinct AFTER understanding the question AND after you have gone through the process of elimination. Without a doubt, examiners of these types of exams will include outrageous choices that will stand out to be wrong. You will more than likely be left with two possible answers, choose based on your first impulse. The first choice would naturally be based on what you have learned, studied, and reviewed. The second choice would be based on a deeper reflection that legal minds are known for: arguing for the other side. Stick with the basic and, chances are, you will get it right. For Definition or Short Answer questions, make sure to use the time as allotted. There is always a tendency to use more time in case you really know the answer. For example, the question was: “What is a holographic will?” Some answers ‘would start by saying, “According to Article 804, 805 and 810 of the New Civil Code...” while others may define what a will is (a will is an act whereby a person is permitted, with the formalities prescribed by law, to control to a certain degree the disposition of this estate, to take effect after his death) before defining a holographic will. It is a more complete answer, but again, the time limitation in the exam may not give you this luxury. A concise answer would be “A holographic will is a will that is written, dated, and signed by the hand of the testator himself. It need not be witnessed, subject to no other form, and can be made in or out of the Philippines.” For Enumeration questions, most of my classmates used mnemonic aids to help them remember a laundry list of grounds for this and that. When I took the Bar exams, there was an enterprising young lawyer who made a business out of selling her old notes to Bar examines. At a cheap price of P100 (I think), her compilation of assorted of mnemonic aids organized by Bar subject and printed and reproduced in mimeographing paper sold like hotcakes! I am not an avid fan of such Enumeration questions since they simply test the students’ memory skill instead of their analytical skill. Then again, some professors would give you these enumeration questions to help you remember fundamental concepts (hopefully) needed for the bigger test, the Bar examinations. ‘8TH BOOSTER: CONQUER ALL OTHERS! a1 (TrueFalse, Multiple Choice, the Bar Exams) 8th Booster ARTICLE: Breaking the Bar ‘Your preparation for the Bar exam started on the first day of your first year class in the College of Law. Almost all of your answers will come from your stock knowledge, from the query that you failed to answer during recitations as well as from those that you were able to answer; from the answers given by your classmates or from those that your professor failed to explain but you had the diligence to research because you were intrigued. Your Bar review for six months or so merely updated you with new cases, refreshed your memory, and boosted your ‘confidence. Your performance in the College of Law mostly and your diligence in the review partly, will spell the difference on your capacity to pass the Bar. Preparing for the Bar Examinations is probably the most difficult mental task you will ever undertake in your life. But the preparation is no different from what I recommended in this book. Of course, it is the biggest and most obvious hurdle for any law student to become a lawyer. You have to give it your best shot. You are on your own during that four Sundays in September. During the review period, you will receive a lot of “unsolicited” advice from many people. Listen to them, but follow your own judgment as to what is the best approach or preparation. _ Be mentally prepared. Due to time constraints, prioritize your reading. If time permits, read first the more recent and ‘important materials. Do not bury yourself in useless material. ‘Bar examinees are “suckers” for buying all the review materials they can get their hands on. They end up buying tons of materials which they will never get to read anyway. Do not feel insecure if the person next to you during the review has a few more boxes of materials than you do. Make an outline of the topics you have to cover and check this against the syllabus of the bar exam. Here are other suggestions: | 1. Review the syllabus or coverage of the exams. 2, Prepare a timetable in terms of number of hours per review material for your review. Regularly check your progress against your timetable. Should you fall behind schedule, just calmly try to catch up. Panic is your worst enemy. 92 8 PERFORMANCE BOOSTERS TO CONQUER ANY LAW EXAM 8. Choose a good reviewer containing all the salient points and covering all of the topics of a bar subject may be a better alternative. Once you have chosen a good reviewer, don't change it. 4, Make sure you have a compilation of the latest Supreme Court decisions. Many Bar exam questions are based on recent cases decided by the Supreme Court. = 6. Do not make the material more complicated than it really is. Itis quite paradoxical that 80% to 90% of the questions in what is supposedly the most difficult law exam are on basic matters. 6 Memorize what you cannot understand! If you encounter material which may seem impossible to comprehend, Just memorize it. No matter what others say, it is possible to memorize even things you do not completely understand. ‘T, _Tape the lectures and listen to them during your non-reading hours. Listen to the tapes even while doing something else. Hearing the material is a very effective way of reinforcing what you have read. When my Wills professor made us memorize Article 805 of the Civil Code (formalities of wills), I managed to memorize it by simply putting on a cassette tape of Article 805 in my car stereo as I traversed the ever-busy traffic along EDSA. While others listened to 89.9 or 99.5, I listened to Article 805! Be physically fit to survive the rigors of the review and of the bar exams. Continue your regular exercise routine. If you do not have any regular exercise, start one. You must exercise! ‘The fatigue and tension you will experience during the review can wear you down fast, both physically and mentally. Be sure that you also get sufficient rest and recreation. You have to give your body and your mind the opportunity to recover. Recreation is equally important to temporarily “divert” your mind from serious study and fight “psychological” and mental tiredness. Get rid of all your emotional problems immediately! Whatever emotional baggage you have, take it off! If you have a quarrel with your parents, siblings, spouse, children, fiancée, ‘8TH BOOSTER: CONQUER ALL OTHERS! 53 (True/False, Multiple Choice, the Bar Exams) friend, classmate, or neighbor, beg for a truce and ask to resume hostilities during the first week of October. Be at peace with the Man above. Ask for Divine Intervention. Although you are responsible for passing, you cannot do it alone. No matter how good you think you are, you can not-pass the bar exams without His help (or that of the supernatural being you think is responsible for the universe). Psyche yourself. Stand in front of the mirror every morning and tell yourself a hundred times: “I will become a lawyer, I will pass the Bar!” Do not be afraid to dream. Remember that for a dream to come true, you must first have a dream. At the end of the review period, you will know everything you need to know to take on the Bar exams and answer whatever questions are asked. ‘Now you are ready to take on the biggest exam of your life. ‘There are just a few more things that you may find helpful. 1. Be punetual: Be on time, or better yet, be early in going to the venue of the Bar Examinations. Do not give yourself the additional pressure by being late. When you are early, you have time to find your room assignment, acquaint yourself with your fellow candidates and relieve yourself of any discomfort before the exams begin. When I took the Bar exams in California, the person sitting next to me arrived just in time for the proctor instructions. I vividly remembered his name since he attracted a lot of attention by coming in late and even reeking with alcohol! I don't have to tell you if his name was on the Pass List released 3 months after the exam. 2. Apply the technique I suggested in answering the questions: There are questions in the Bar exams that you will find easy so that you will be confident in answering them at once. In objective questions, there should be no problem because your memory will take care of this type of question. However, for essay questions, follow the technique I recommended in the first part of this book. Strictly observe NEATNESS, CORRECT SPELLING and GRAMMAR and MARGINS in answering the questions. Remember that of all exams, the Bar exams involve not only the substance of your answer but also the PRESENTATION of your ANSWERS! When you want ‘54 8 PERFORMANCE BOOSTERS TO CONQUER ANY LAW EXAM to erase an answer, simply place a horizontal line across the answer desired to be stricken off and bracket the portion then start your answer in the next paragraph. You will hear a lot of ghastly stories, some already folklore, regarding other people's experiences in writing the Bar exam. Be not afraid, compose yourself and take the Bar exam as if it were any other law ‘school exam. 3. When you finish the examination given in the morning, give your mind some rest and enjoy your lunch. Read your notes for the examination in the afternoon only when you have the time. Never discuss your answers in the morning ‘exam with your fellow Bar candidates because you would either feel high or low but most of the time you will be discouraged especially if your answers were not the same as the others. Comparing answers will only disrupt the focus you need for the next subject. 4. Finally, never write in your bluebook something like this: “Please grant my wish to become a lawyer before I die”. In all likelihood, it will be considered a marking and you will be disqualified. The Bar examination may decide your fate as to whether ‘you will have that title “Attorney” prefixed in your name; but, Just the same it is just one of those law exams that can be ‘conquered using the Performance Boosters in this book. Summary ‘The best approach to any law exam, to inelude the Bar examinations, is to believe in yourself. As long as you put in the time during law school, you are more than equipped to pass ANY and ALL law exams! Prior to the examination, prepare yourself mentally, physically, emotionally, spiritually, and psychologically. ‘Manny Pacquiao will tell you that he wins most of his matches even before the bell of the first round, He prepares and trains so hard that on the day of the fight, every thing falls into place. No second guessing, it will be a matter of execution of your game plan. So, what is your game plan for any law examination? ‘Just remember this process: Conquer your fear with a positive approach towards passing the exam, analyze the fact pattern, spot the issues, make an outline, then write beautifully! ‘This is how you can pass any and all law exam(s) that ‘you will take for the rest of your life. It is not a guarantee but it certainly worked for me, my classmates, and a lot of my students. Bonus: Sample Question, Analysis, and Answer LONG ESSAY: Below is an actual question and suggested answer given in February 2005 California Bar exams. Maria and Juana were partners in the operation of a coin laundry store. Recent environmental regulations relating to dangers posed by detergents increased their exposure to Viability and caused a decline in their business. Maria and Juana decided to convert their partnership into Lavaco, Inc. Lavaco”), a corporation, to limit their potential personal liability. Maria and Juana each contributed P2 M in cash to Lavaco. In return, each received a P1.5M promissory note from Lavaco and 5,000 shares of stock with a value of P100 per share. Prior to incorporation, Maria entered into a contract on behalf of Lavaco with Kula Company (“KC”) for the credit purchase of an environmentally detergent maker for P10 M. KC was aware that Lavaco had not yet been formed. KC delivered the detergent maker one week after the incorporation, and Lavaco used it thereafter and made monthly installment payments. Lavaco had been incorporated in compliance with all SEC requirements, and Maria and Juana observed all corporate formalities during the period of Lavaco’s existence. One year after incorporation, however, Lavaco became insolvent and dissolved. At the time of the dissolution, Lavaco's assets were valued at P500,000. Its debts totaled P12 M, consisting of the two P1.5 M notes held by Maria and Juana and a P9M balance due KC for the patented detergent maker. As among KC, Maria, and Juana, how should Lavaco's 'P500,000 in assets be distributed? Discuss. SUGGESTED ANSWER Distribution of Lavaco's 500,000 in Assets 66 ‘BONUS: SAMPLE QUESTION, ANALYSIS, AND ANSWER 57 Valid De Jure Corporation A corporation is conclusively formed when the articles of incorporation are filed in accordance with the law governing corporations. Here, the facts indicate that Lavaco had been incorporated in compliance with all statutory compliances. ‘Therefore, Lavaco will be treated as a de jure corporation. ‘The Kula Company Contract (KC) Whether KC will have a claim to Lavaco’s assets on dissolution depends on whether KC’s pre-incorporation contract with Maria as a promoter was adopted by Lavaco. ‘A corporation is not liable for pre-incorporation contracts unless the corporation adopts the contract. Since Lavaco did not exist at the time the contract was made, it can have liability unless the corporation expressly adopts the contract through board resolutions for instance or the corporation accepts or retains benefits from the contract and therefore impliedly adopts the contract. On these facts, Lavaco accepted the detergent maker, used it, and made monthly payments on it. Even though KC was aware that Lavaco had not yet been formed, Maria entered the contract on Lavaco’s behalf. Further the detergent maker was delivered after incorporation. KC will argue that Lavaco’s acceptance and use of the detergent maker constitutes implied adoption, and will likely prevail. Therefore, KC has a valued unsecured claim against Lavaco’s assets. Promissory Note Promissory Notes are debt securities of a corporation. The holders of these notes have a creditor/debtor relationship with the corporation, and are on equal grounds with other unsecured creditors of the corporation. Shareholders’ Claims Shareholders own an equity interest in a corporation. Shareholders are not entitled to distribution of a dissolved corporation's assets until all debts of the corporation have been satisfied. Distribution KC and Maria and Juana stand on equal footing as unsecured creditors. As shareholders, Maria and Juana will 58 8 PERFORMANCE BOOSTERS T0 CONQUER ANY LAW EXAM receive no part of the $50K, as explained above. As between unsecured creditors, however, there is a possibility that ‘Maria/Juana’s claim will be subordinated by a court to KC’s claim, based on corporate veil piercing principals [sic] due to inadequate capitalization at the outset of the corporation. Piercing the Corporate Veil A corporation is a separate legal entity designed to insulate its officers, directors, and shareholders from personal liability. However, the corporate form will be ignored in some circumstances, including when the corporation is acting as the alter ego of the shareholders or when there was inadequate capitalization of the corporation at the outset. Inadequate capitalization is determined by looking at if the corporation had adequate funds to meet its prospective liabilities. The time between incorporation and dissolution is also considered. Here, Lavaco was funded with P4M and was dissolved within one year. The short time in existence maybe an indication that the corporation was not adequately funded. However, it is unclear from these facts what caused Lavaco's dissolution. If Maria/Juana were aware of increasing environmental costs and liability, P4M may not have been sufficient. If this is so the corporate veil will be pierced. (Desire to shield from personal liability from environmental regulation is not enough to pierce the veil in and of itself.) When shareholders use the corporate assets as their own or otherwise ignore corporate formalities, the corporate form may be ignored to hold the shareholders personally liable for corporate debts, Here, there is no indication ‘that Juana/Mary used Lavaco’s assets as their own and they did observe all corporate formalities. Therefore, the veil will not be pierced on this theory. Since the veil can be pierced due to inadequate capitalization, however, Juana/Mary’s claim on the unsecured notes will be subordinated to KC's claim, KC will receive the entire $P500,000. In the event the claims are not subordinated, KC, Mary and Juana will equally divide the P500,000.

Вам также может понравиться